PDA

نسخه کامل مشاهده نسخه کامل : ◄◄ اتــاق اثبــات فــرمــول ها،قــضــایــا و احــکام هــنــدســه ►►



صفحه ها : 1 [2]

IaHM
22-11-2010, 14:06
--------------------------------------------------------

ali_hp
23-11-2010, 18:17
سلام
یه سوال قدیمی هندسه هنوزهم توی مغزم اذیت میکنه نتونستم تا الان حلش کنم !
میگه که یه مثلثی رو رسم کنید که فقط اندازه ی سه ارتفاعش معلوم باشه !
روش رسمش ؟

سلام
می تونین از قضیه زیر کمک بگیرین:
مثلث دلخواهی مفروض است،مثلثی که طول اضلاعش برابر معکوس ارتفاعات مثلث مفروض باشد،با مثلث مفروض متشابه است و نسبت تشابه 2S است که در آن S مساحت مثلث مفروض است.

اثباتشم واضحه،همون فرمول مساحت مثلث بر حسب ضلع و ارتفاع وارد به ضلعه!

davy jones
30-11-2010, 20:55
سلام دوستان. من به یه سوال سخت برخوردم که حقیقتا تو حلش موندم. گفتم شاید سایر دانشمندان عزیز کمکی بکنن.

سوال: ثابت کنید در هر مثلث دلخواه، 4 نقطه ای که به ترتیب نقاط همرسی میانه ها، عمودها، عمود منصفها و زاویه های داخلی آن مثلث است؛ همواره در امتداد یک خط راست قرار دارند (برای درک بهتر از سوال به شکل زیر نگاه کنین)


[ برای مشاهده لینک ، لطفا با نام کاربری خود وارد شوید یا ثبت نام کنید ]


موفق باشین.
89/9/9

ALt3rnA
02-12-2010, 20:35
سلام دوستان. من به یه سوال سخت برخوردم که حقیقتا تو حلش موندم. گفتم شاید سایر دانشمندان عزیز کمکی بکنن.

سلام
اگه لطف کنی و بگی چطوری میتونم شکل هندسی دقیق بکشم برات سعی میکنم حلش کنم بزارم

davy jones
02-12-2010, 21:04
سلام
اگه لطف کنی و بگی چطوری میتونم شکل هندسی دقیق بکشم برات سعی میکنم حلش کنم بزارم

سلام.

از این سایت استفاده کنین. به صورت آنلاین میشه توش اشکال دقیق هندسی کشید:


برای مشاهده محتوا ، لطفا وارد شوید یا ثبت نام کنید

موفق باشین.
89/9/11

lebesgue
03-12-2010, 02:23
سلام دوستان. من به یه سوال سخت برخوردم که حقیقتا تو حلش موندم. گفتم شاید سایر دانشمندان عزیز کمکی بکنن.

سوال: ثابت کنید در هر مثلث دلخواه، 4 نقطه ای که به ترتیب نقاط همرسی میانه ها، عمودها، عمود منصفها و زاویه های داخلی آن مثلث است؛ همواره در امتداد یک خط راست قرار دارند (برای درک بهتر از سوال به شکل زیر نگاه کنین)


[ برای مشاهده لینک ، لطفا با نام کاربری خود وارد شوید یا ثبت نام کنید ]


موفق باشین.
89/9/9



البته خط مورد نظر از نقطه همرسی نیمساز زوایای داخلی نیمگذره الزاماً.

یک اثبات ساده برای اون سه نقطه دیگه به این صورت هست.

فرض کنید O نقطه همرسی عمود منصفها و G نقطه همرسی میانه های مثلث ABC باشد.
OG را تا نقطه P امتداد می دهیم بطوریکه داشته باشیم OG/GP = 1/2 .
نشان می دهیم P نقطه همرسی عمودها است.
میانه 'AA که در آن 'A نقطه میانی BC است را رسم می کنیم.
مثلث های 'OGA و PGA متشابه هستند، زیرا GP=2GO و AG=2A'G و همچنین زوایای 'OGA و PGA برابرند.
پس زوایای OA'G و PAG برابرند و در نتیجه AP و 'OA موازی هم هستند.
از طرفی 'OA عمود بر BC است، در نتیجه AP نیز عمود بر BC خواهد بود که این یعنی P روی ارتفاع رسم شده از A بر BC قرار دارد.
با تکرار همین استدلال برای دو میانه دیگر، ثابت می شود که P روی دو ارتفاع دیگر نیز قرار دارد، در نتیجه نقطه همرسی ارتفاع ها است.
[ برای مشاهده لینک ، لطفا با نام کاربری خود وارد شوید یا ثبت نام کنید ]

کپی برداری شده از:

برای مشاهده محتوا ، لطفا وارد شوید یا ثبت نام کنید

البته گویا این خط (که به خط اولر معروف هست) از چندین نقطه مهم دیگه مثلثها هم میگذره:

برای مشاهده محتوا ، لطفا وارد شوید یا ثبت نام کنید

saeed_cpu_full
12-12-2010, 19:11
سلام دوستان
یه سؤال چند وقت پیش دیدم ولی هر کاری کردم نتونستم ثابت کنم .... اگه میشه دوستان کمک کنند

در ذوزنقه ی ABCD ، نقطه ی E وسط قطر DB و F وسط قطر AC است. از F عمود FK را بر ساق BC و از E عمود ET را بر ساق AD رسم میکنیم. ثابت کنید نقطه ی برخورد این دو عمود روی یکی از عمود منصف های یکی از قاعده های ذوزنقه است.

aidauka
28-12-2010, 14:51
لطفا هر چه زودتر اثبات هاي فيثاغورس براي من ارسال كنيد:20:

davy jones
28-12-2010, 15:09
لطفا هر چه زودتر اثبات هاي فيثاغورس براي من ارسال كنيد:20:
خودتون میتونین از اینجا ([ برای مشاهده لینک ، لطفا با نام کاربری خود وارد شوید یا ثبت نام کنید ]) ملاحظه بفرمائین.

موفق باشین.
89/10/7

mbsh
10-02-2011, 00:36
سلام دوستان
میشه یکی به من کمک کنه. می خوام پیدا کنم، چه جوری یه هرم 4 وجهی به یک مکعب 6 وجهی می تونه تبدیل بشه. هم تصورش و مفهومشو اگه کسی بتونه برام توضیح بده و هم فرمولای تبدیلشو، خیلی ازش ممنون میشم.

davy jones
10-02-2011, 11:05
سلام دوستان
میشه یکی به من کمک کنه. می خوام پیدا کنم، چه جوری یه هرم 4 وجهی به یک مکعب 6 وجهی می تونه تبدیل بشه. هم تصورش و مفهومشو اگه کسی بتونه برام توضیح بده و هم فرمولای تبدیلشو، خیلی ازش ممنون میشم.
سلام. منظورتونو واضح تر بگین. یعنی چی که تبدیل میشه؟ هرم آخه چه ربطی داره به مکعب؟

mbsh
10-02-2011, 19:40
سلام
این به ذهن خودم رسید. البته بدون فرمول:
1.هرم با قاعده مربع را تصور می کنیم.حال اگر بخواهیم آن به یک مکعب تبدیل
کنیم: می دانیم اگر راس هرم را بر قاعده اش تصویر کنیم دقیقا در ئسط آن
می افتد. پس برای تبدیلش به مکعب، مثلا اگر قرار است تبدیل همراه با
انتقال به اندازه x باشد،ابتدا نقاط قاعده هرم را به همان اندازه x
انتقال می دهیم. سپس راس هرم را هم دقیقا به همان اندازه x انتقال می
دهیم. حال با اتصال نقاط انتقال یافته از قاعده که قاعده مکعب حاصل می
شود. و اما برای وجه بالایی مکعب: نقطه حاصل از انتقال راس هرم را از 4
جهت راست و چپ و بالا و پایین به اندازه نصف ضلع قاعده هرم و به موازات
قاعده حاصل از انتقال ادامه داده و به نقاط جدیدی می رسیم. حال این نقاط
را به همان موازات اضلاع مکعب و از 2 طرف به همان اندازه نصف ضلع قاعده
هرم، ادامه می دهیم و نقاط حاصل از تلاقی ها، رئوس مکعبند که با اتصالشان
به هم و سپس اتصال هر نقطه به نقطه متناظرش در قاعده مککع، ارتفاع مکعب
هم رسم شده و مکعب حاصل از تبدیل هرم تکمیل می شود.
نمی دانم خوب توضیح دادم یا نه. ولی خودم دقیقا متوجه شدم که در این
تبدیل چه باید بشود و البته فرمول خاصی جز همان فرمول انتقال به میزان x
و نیز نصف ضلع قاعده هرم را لازم نداریم!
2.برای تصور بهتر حرکت معکوس یعنی، ساخته شدن هرم از مکعب را در نظر می
گیریم. مکعب را با وجوه مربعی با ضلعa در نظر می گیریم. حال اگر قطر هر
وجه را ترسیم کنیم، هرم با قاعده مثلثی حاصل می شود. در نتیجه اندازه
اضلاع هرم طبق قضیه فیثاغورس برابر می شود با رادیکال 2 ضرب در a به
توان 2. برای تبدیل معکوس این حالت، می توانیم هرمی با اندازه اضلاع به
دست آمده را به مکعبی به ضلع a تبدیل کرد. و سپس در صورت نیاز، با تبدیل
تجانس، اندازه مکعب را به میزان دلخواه تبدیل کرد.

kasra_khan2003
28-02-2011, 22:42
سلام، آیا این رابطه برقرار هست؟ هر دو طرف هم علامت هستن.


برای مشاهده محتوا ، لطفا وارد شوید یا ثبت نام کنید

davy jones
01-03-2011, 08:49
سلام، آیا این رابطه برقرار هست؟ هر دو طرف هم علامت هستن.


برای مشاهده محتوا ، لطفا وارد شوید یا ثبت نام کنید
سلام.
این رابطه همواره که برقرار نیست. در ثانی ما نمیدونیم a اصلا چه مقداری داره. یه پرانتز هم انگار جا افتاده.

اگهبه نظرتون من مظورتونو نتونستم هنوز بفهمم، لطفا بیشتر توضیح بدین. برای تایپ آسان و حرفه ای فرمولهای ریاضی هم به لینک موجو در امضام مراجعه کنین.

موفق باشین.
89/12/10

mj425
04-03-2011, 21:10
سلام
کسی میدونه فرمول تاب منحنی چه جوری اثبات میشه؟ لطفا فوری راهنمایی کنید!:13:
اینم فورمولش: [ برای مشاهده لینک ، لطفا با نام کاربری خود وارد شوید یا ثبت نام کنید ]

nimamozi
11-03-2011, 14:56
دوستان سلام :18:.
اثبات این فرمول رو هر چه زودتر هر کسی که تونست برام بذاره دعاش میکنم:

a^2=b^2+c^2-2bc*cosA

sarah.A
01-04-2011, 22:19
سلام
میشه کمکم کنید؟؟؟:46:
یه سوال دارم
ثابت کنید:
اگر دو نیمساز داخلی مثلثی مساوی باشند آن مثلث متساوی الساقین است.
نمی دونم از چه راهی برم :41:

ali_hp
07-04-2011, 17:59
سلام
میشه کمکم کنید؟؟؟:46:
یه سوال دارم
ثابت کنید:
اگر دو نیمساز داخلی مثلثی مساوی باشند آن مثلث متساوی الساقین است.
نمی دونم از چه راهی برم :41:
سلام
اینجا حل شده:

برای مشاهده محتوا ، لطفا وارد شوید یا ثبت نام کنید
البته سوال ساده ای نیست!

dDorsa
12-05-2011, 00:03
سلام من جواب سوالهای شمارو نمی دونم ولی خودم به یه مشکلی برخورد کردم امکانش هست کمکم کنید؟:11:

dDorsa
12-05-2011, 00:09
سلام به همگی من اطلاعات کمی در مورد فیزیک دارم .الانم به یه مشکلی بر خوردم می شه کمکم کنید؟:11:

davy jones
13-05-2011, 15:55
سلام من جواب سوالهای شمارو نمی دونم ولی خودم به یه مشکلی برخورد کردم امکانش هست کمکم کنید؟[ برای مشاهده لینک ، لطفا با نام کاربری خود وارد شوید یا ثبت نام کنید ]



سلام به همگی من اطلاعات کمی در مورد فیزیک دارم .الانم به یه مشکلی بر خوردم می شه کمکم کنید؟:11:
به جای ارسال اسپم سوالتون رو اینجا کامل و دقیق بنویسید و منتظر جواب باشین. اگه کسی بلد بود به شما کمک میکنه. ضمنا اینجا محل پرسش و پاسخ و بحث در مورد زمینه های مختلف ریاضیاته و قرار نیست سوالات فیزیک در اون پرسیده بشه.

موفق باشین.
90/20/23

saw fish
21-06-2011, 10:02
ببخشین ممکنه یک سایت برای کشیدن اشکال هندسی معرفی کنین

همچنین می خواستم اثبات کنین که در مثلث قائم الزاویه ضلع روبرو به زاویه 30 درچه نصف وتر است

mofidy1
21-06-2011, 13:09
ببخشین ممکنه یک سایت برای کشیدن اشکال هندسی معرفی کنین

همچنین می خواستم اثبات کنین که در مثلث قائم الزاویه ضلع روبرو به زاویه 30 درچه نصف وتر است

با سلام

برای این کار به سایت geogebra با آدرس زیر مراجعه کنید. توجه بفرمایید که باید روی رایانه ی خودتان جاوا نصب کرده باشید.


برای مشاهده محتوا ، لطفا وارد شوید یا ثبت نام کنید

درباره ی مساله ی دوم، دقت کنید که سینوس 30 درجه برابر است با «یک دوم» بنابر این نسبت ضلع روبه رو به زاویه ی 30 درجه به خود وتر «یک دوم» است، یعنی ضلع روبرو به زاویه 30 درچه نصف وتر است.

موفق باشید.

31 خرداد 1390

saw fish
23-06-2011, 10:17
برای رسم اشکال هندسی غیر از این سایت، سایت دیگری سراغ ندارین؟

ببخشید اگر یک سایت هم برای نوشتن فرمول های ریاضی پیدا کردین لطفا خبر بدین

ankel
28-10-2011, 16:40
یه سوال دیگه هم داشتم
فرم دنباله ای این رشته اعداد رو میخوام... (مقصودم به - و + شدن یکی در میان اعداد هست که نمیدونم چه جوری باید مشخص کنم) :13:
2و 4-و 6و 8-و 10و 12-و...
kheli sadas benevis 2n zarndar 1 be tavane n+1 ke n shomare jomlas
:31::10:

hamed6672
28-10-2011, 18:39
kheli sadas benevis 2n zarndar 1 be tavane n+1 ke n shomare jomlas
:31::10:


البته اشتباه تایپ کردین 1 نه 1-

:46:

mona jooooooooooon
12-12-2011, 15:51
سلام من ترم اولي نرم افزارم لطفا كمك كنيد
اثبات فرمول جايگشت ترتيب و تركيب رو
فرمولشو اينجا ميزنم خراب مينويسه
اگه كسي بلده كمك كنه
تازه 2 جلسه اس كه سركلاس ميريم بعدا يه چيزايي ازمون ميخوان در حد المپيك

davy jones
27-02-2012, 17:13
سلام.

یه سوال نسبتا متوسط مطرح میکنم تا علاقه مندان جواب بدن.
بعد از حدود یک هفته اگه کسی جواب نداد، راه حل رو قرار میدم.

سوال:

در مثلث ABC داریم: [ برای مشاهده لینک ، لطفا با نام کاربری خود وارد شوید یا ثبت نام کنید ] &space;%5Cleft&space;%7CBC&space;%5Cright&space;%7C=d
و در مثلث 'A'B'C داریم: [ برای مشاهده لینک ، لطفا با نام کاربری خود وارد شوید یا ثبت نام کنید ] ,%5C;&space;%5C;&space;%5Cleft&space;%7C%7BB%7D%27%7BC%7D%27&space;%5Crigh t&space;%7C=x

به ازای چه مقداری از x بر حسب [ برای مشاهده لینک ، لطفا با نام کاربری خود وارد شوید یا ثبت نام کنید ] و d ، میتوان با قاطعیت گفت که مثلث 'A'B'C را نیز میتوان در دایره ی محیطی مثلث ABC ، محاط کرد؟؟


موفق باشین.
90/12/8

davy jones
04-03-2012, 09:53
سلام.

یه سوال نسبتا متوسط مطرح میکنم تا علاقه مندان جواب بدن.
بعد از حدود یک هفته اگه کسی جواب نداد، راه حل رو قرار میدم.

سوال:

در مثلث ABC داریم: [ برای مشاهده لینک ، لطفا با نام کاربری خود وارد شوید یا ثبت نام کنید ] &space;%5Cleft&space;%7CBC&space;%5Cright&space;%7C=d
و در مثلث 'A'B'C داریم: [ برای مشاهده لینک ، لطفا با نام کاربری خود وارد شوید یا ثبت نام کنید ] ,%5C;&space;%5C;&space;%5Cleft&space;%7C%7BB%7D%27%7BC%7D%27&space;%5Crigh t&space;%7C=x

به ازای چه مقداری از x بر حسب [ برای مشاهده لینک ، لطفا با نام کاربری خود وارد شوید یا ثبت نام کنید ] و d ، میتوان با قاطعیت گفت که مثلث 'A'B'C را نیز میتوان در دایره ی محیطی مثلث ABC ، محاط کرد؟؟


موفق باشین.
90/12/8
سلام.

یک هفته گدشت و کسی جواب نداد! :13:

و اما جواب:

در تعریف کمان درخور یک پاره خط دلخواه مانند AB به طول d داریم:

کمان درخور پاره خط AB ، مکان هندسی همه ی نقاطی است که از آنها پاره خط AB به زاویه ی [ برای مشاهده لینک ، لطفا با نام کاربری خود وارد شوید یا ثبت نام کنید ] دیده می شود:


[ برای مشاهده لینک ، لطفا با نام کاربری خود وارد شوید یا ثبت نام کنید ]


که این مکان هندسی طبق تعریف زاویه ی محاطی، برابر با کمانی از یک دایره است که پاره خط AB وتری از آن داریره خواهد بود.
همچنین طبق تعریف زاویه ی محاطی و زاویه ی مرکزی در دایره، زاویه ی مرکزی یک کمان همواره 2 برابر زاویه ی محاطی همان کمان است. بنابراین زاویه ی M برابر با [ برای مشاهده لینک ، لطفا با نام کاربری خود وارد شوید یا ثبت نام کنید ] است. اگر در مثلث ABM که متساوی الساقین هم هست (چرا؟) نیمساز راس M را رسم کنیم به پاره خط AB عمود میشود. بنابراین شعاع دایره حاوی کمان درخور (که همان دایره ی محیطی مثلث اصلی مساله هم هست) این چنین بدست میآید:


[ برای مشاهده لینک ، لطفا با نام کاربری خود وارد شوید یا ثبت نام کنید ] Cfrac%7B%7CAs&space;%7C%7D%7BR%7D%5CRightarrow&space;R=%5Cfrac %7B%7CAs&space;%7C%7D%7B%5Csin&space;%28%5Calpha%29%7D=%7B%5Cc olor%7BRed%7D&space;%5Cfrac%7Bd%7D%7B2%5Csin&space;%28%5Calpha &space;%29%7D%7D


بنابراین با دانستن مقادیر یک زاویه از مثلث و طول ضلع مقابل به آن زاویه میتوان شعاع دایره ی محیطی را در آن دایره محاسبه کرد.

با این مقدمه به سراغ دو مثلث اشاره شده در صورت سوال میریم:

در مثلث ABC شعاع دایرهی محیطی برابر است با:


[ برای مشاهده لینک ، لطفا با نام کاربری خود وارد شوید یا ثبت نام کنید ] sin&space;%28%5Calpha&space;%29%7D


همچنین در مثلث 'A'B'C شعاع دایره ی محیطی برابر است با:


[ برای مشاهده لینک ، لطفا با نام کاربری خود وارد شوید یا ثبت نام کنید ] 7D%7B2%5Csin&space;%282%5Calpha&space;%29%7D


و اگر طول این دو شعاع با هم برابر باشند خواهیم داشت:


[ برای مشاهده لینک ، لطفا با نام کاربری خود وارد شوید یا ثبت نام کنید ] 5Calpha%29%7D%7B%5Csin&space;%28%5Calpha%29%7Dd=%7B%5Cco lor%7BGolden%7D&space;2d.%5Ccos&space;%28%5Calpha%29%7D



سطح این سوال در حد معلومات هندسه ی سوم دبیرستان یود :46:



موفق باشین.
90/12/14

skyzare
04-03-2012, 13:01
با سلام .

با تشکر از شما .

جالب بود . :20:



یک هفته گدشت و کسی جواب نداد! :13:


خوب سخت بود .:31:حداقل برای من یکی که پست رو دیدم شاید بعضی ها هم پست رو ندیدند .


اگر در مثلث ABM که متساوی الساقین هم هست (چرا؟)

بفرمایید این هم پاسخ چرای شما :

چون دو ساق شعاع دایره هستند .

amaaa
10-03-2012, 18:07
لطفا اثبات روابط مثلثاتی را بگذاری؟

nazanin_75
27-04-2012, 12:27
سلام لطفا اگر می تونید این سوال منو با راه حل کامل واسم ایمیل کنید...........
اگر در داخل یک مثلث متساوی الاضلاع به ضلع a دایره ای محاط کنیم سپس در درون آن دایره ,مثلث متساوی الاضلاع دیگری محاط کنیم و این کار را تا بی نهایت بار ادامه دهیم...
الف) مجموع مساحت مثلث ها؟؟؟؟؟؟؟
ب)مجموع مساحت دایره ها؟؟؟؟؟؟؟
ج) اگر این کار را با یک n ضلعی انجام دهیم مجموع مساحت دایره ها و مجموع مساحت مثلث ها چند خواهد شد؟؟؟؟؟؟
د)آیا رابطه ای میان مساحت دایره و n ضلعی ها وجود دارد؟؟؟؟؟؟؟

davy jones
13-06-2012, 17:36
سلام.

یه مساله ی جالب دیدم گفتم اینجا مطرحش کنم تا دوستان در موردش نظر بدن:

سوال: مکان هندسی کانونی از یک بیضی، که در آن بیضی، کانون دیگر و 2 نقطه از محیط آن بیضی همواره در صفحه ثابت اند، چیست؟


اگه تا یک هفته کسی جواب نداد، جواب رو میذارم.


موفق باشین.
91/3/24

lebesgue
19-06-2012, 17:17
بنا به تعریف، بیضی مکان هندسی نقاطی از صفحه است که مجموع فواصلشان از دو نقطه ثابت (کانونها)، ثابتی مثبت است. پس اگر A و B دو نقطه روی بیضی باشند که فاصله A از کانون اول، d واحد کمتر از فاصله B از کانون اول باشد، آنگاه فاصله A از کانون دوم، d واحد بیشتر از فاصله B از کانون دوم خواهد بود. پس مکان هندسی کانون دوم، مجموعه نقاطی از صفحه است که فاصله شان از نقطه A، به مقدار d واحد بیشتر از فاصله شان از نقطه B باشد، که این همان تعریف هذلولی است. البته تنها یک شاخه اش.

davy jones
20-06-2012, 23:59
بنا به تعریف، بیضی مکان هندسی نقاطی از صفحه است که مجموع فواصلشان از دو نقطه ثابت (کانونها)، ثابتی مثبت است. پس اگر A و B دو نقطه روی بیضی باشند که فاصله A از کانون اول، d واحد کمتر از فاصله B از کانون اول باشد، آنگاه فاصله A از کانون دوم، d واحد بیشتر از فاصله B از کانون دوم خواهد بود. پس مکان هندسی کانون دوم، مجموعه نقاطی از صفحه است که فاصله شان از نقطه A، به مقدار d واحد بیشتر از فاصله شان از نقطه B باشد، که این همان تعریف هذلولی است. البته تنها یک شاخه اش.

سلام.

با تشکر از جناب 1233445566 ([ برای مشاهده لینک ، لطفا با نام کاربری خود وارد شوید یا ثبت نام کنید ]) که به درستی و با توضیحی رسا و کامل، جواب رو گفتند.

یه سوال دیگه هم در همین رابطه مطرح میکنم که مشابه قبلیه:

سوال: در ذوزنقه ی قائم الزاویه [ برای مشاهده لینک ، لطفا با نام کاربری خود وارد شوید یا ثبت نام کنید ] داریم:

1- [ برای مشاهده لینک ، لطفا با نام کاربری خود وارد شوید یا ثبت نام کنید ] D%7B2%7D

2- [ برای مشاهده لینک ، لطفا با نام کاربری خود وارد شوید یا ثبت نام کنید ]

3- [ برای مشاهده لینک ، لطفا با نام کاربری خود وارد شوید یا ثبت نام کنید ] C&space;%5Cright&space;%7C

4- [ برای مشاهده لینک ، لطفا با نام کاربری خود وارد شوید یا ثبت نام کنید ] ant&space;in&space;surface%7D (نقطه های C و D از ذوزنقه در صفحه ثابت و معین هستند.)

حال سوال اینجاست که مکان هندسی راس B در تمام ذوزنقه هایی که 4 شرط بالا را دارند چیست؟


باز هم تا یک هفته فرصت دارین برای پاسخگویی.


موفق باشین.
91/4/1

davy jones
29-06-2012, 20:34
سلام.

با تشکر از جناب 1233445566 ([ برای مشاهده لینک ، لطفا با نام کاربری خود وارد شوید یا ثبت نام کنید ]) که به درستی و با توضیحی رسا و کامل، جواب رو گفتند.

یه سوال دیگه هم در همین رابطه مطرح میکنم که مشابه قبلیه:

سوال: در ذوزنقه ی قائم الزاویه [ برای مشاهده لینک ، لطفا با نام کاربری خود وارد شوید یا ثبت نام کنید ] داریم:

1- [ برای مشاهده لینک ، لطفا با نام کاربری خود وارد شوید یا ثبت نام کنید ] D%7B2%7D

2- [ برای مشاهده لینک ، لطفا با نام کاربری خود وارد شوید یا ثبت نام کنید ]

3- [ برای مشاهده لینک ، لطفا با نام کاربری خود وارد شوید یا ثبت نام کنید ] C&space;%5Cright&space;%7C

4- [ برای مشاهده لینک ، لطفا با نام کاربری خود وارد شوید یا ثبت نام کنید ] ant&space;in&space;surface%7D (نقطه های C و D از ذوزنقه در صفحه ثابت و معین هستند.)

حال سوال اینجاست که مکان هندسی راس B در تمام ذوزنقه هایی که 4 شرط بالا را دارند چیست؟


باز هم تا یک هفته فرصت دارین برای پاسخگویی.


موفق باشین.
91/4/1

سلام.

بیش از یک هفته گذشت و کسی جواب نداد.

این هم جواب:

طبق شرط شماره ی 3، در ذورنقه ی ABCD فاصله ی نقطه ی B از خط AD همواره برابر با فاصله ی این نقطه از نقطه ی C هستش.
و طبق شرط شماره ی 4، فاصله ی نقطه ی C از خط AD هم همواره مقدار ثابتی هستش.

پس بنابراین اگر خطی رو که از دو نقطه ی A و D میگذره رو از هر دو طرف، امتداد بدیم و این خط رو L بنامیم، این خط همواره در جای خودش، ثابته. از طرفی نقطه ی C هم همواره در بیرون از خط L قرار داره و ثابته.

و در این حالت، نقطه ی B مکان هندسی نقاطیه که از خط L و نقطه ی ثابت C به یک فاصله اند که این دقیقا تعریف یک سهمی است.

برای درک بهتر به شکل سمت راست دقت کنید:


[ برای مشاهده لینک ، لطفا با نام کاربری خود وارد شوید یا ثبت نام کنید ]


موفق باشین.
91/4/9

lebesgue
01-07-2012, 00:52
مثلثهای T1 و T2 در یک صفحه واقع بوده و T2 درون T1 جای دارد. آیا امکان دارد که محیط T2 بزرگتر از محیط T1 باشد؟ درستی پاسخ خود را نشان دهید.

Kesel
01-07-2012, 22:18
مثلثهای T1 و T2 در یک صفحه واقع بوده و T2 درون T1 جای دارد. آیا امکان دارد که محیط T2 بزرگتر از محیط T1 باشد؟ درستی پاسخ خود را نشان دهید.

خیر ممکن نیست.برای توضیح بهتر دو محور مختصات را برای صفحه ی مذکور در نظر می گیریم به طوری که یکی از محور ها موازی یکی از اضلاع مثلث T1 باشد.آنگاه برای رسم مثلث کوچکتر که قرار است درون T1 واقع شود ، باید از اضلاعی استفاده کنیم که هم مولفه ی اول کوچکتر و هم مولفه ی دوم کوچکتر دارند . لذا بنابر قاعده ی فیثاغورس ضلع سوم هم قطعا کوچکتر از ضلع سوم مثلث T1 است . لذا محیط آن (مجموع سه ضلع) کوچک تر خواهد بود.

lebesgue
02-07-2012, 10:34
پاسخ شما را متوجه نمیشوم. آیا منظور شما از مولفه های یک ضلع، طول تصویرش بر محورهای مختصات است؟


باید از اضلاعی استفاده کنیم که هم مولفه ی اول کوچکتر و هم مولفه ی دوم کوچکتر دارند .
این مولفه ها از چه چیزی کوچکترند؟

Kesel
03-07-2012, 09:09
پاسخ شما را متوجه نمیشوم. آیا منظور شما از مولفه های یک ضلع، طول تصویرش بر محورهای مختصات است؟


این مولفه ها از چه چیزی کوچکترند؟

به خاطر تاخیر در پاسخ عذر خواهی می کنم.
خیر منظور من در اصل این گونه است:
توجه داریم که برای یافتن بزرگترین محیط ممکن باید هر سه ضلع بزرگترین مقادیر خود را اختیار کنند.
1 - برای ساده تر شدن مساله یکی از محور ها را (به نام x) منطبق (حالت خاصی از توازی) بر یکی از اضلاع مثلث T1 در نظر می گیریم . هم چنین باز هم برای ساده شدن ، مبدا مختصات را نیز منطبق بر یکی از دو راس سر ضلع مذکور فرض می کنیم.
اکنون بزرگ ترین ضلعی که از نظر مختصه ی اول ممکن است درون این مثلث جای گیرد ، ضلعی است موازی ضلع منطبق بر محور x ها و با اختلاف ناچیز از آن . در هر حال این ضلع ، کوچک تر از ضلع منطبق بر محور x هاست.
2 - در این مرحله باید ضلعی دیگر از مثلث T2 را به طریق مشابه آنالیز کنیم ، لذا این ضلع نیز حداکثر مولفه ی دوم (y) که خواهد داشت ، اندکی کوچکتر از عریض ترین نقطه ی مثلث T1 است . بنابراین ناگزیر این ضلع نیز کوچک تر از ضلعی دیگر از این مثلث است .
3 - از دو قسمت قبل به دست آمد که دو ضلع مثلث T2 کوچک تر از دو ضلع مثلث T1 است . بنابر قاعده ی فیثاغورس ضلع سوم نیز کوچک تر است . لذا مجموع سه ضلع T2 کوچک تر از مجموع سه ضلع T1 است.

lebesgue
10-07-2012, 12:20
از پاسخ شما سپاسگزارم.



توجه داریم که برای یافتن بزرگترین محیط ممکن باید هر سه ضلع بزرگترین مقادیر خود را اختیار کنند.

این گزاره کمی ابهام دارد. اگر طول بزرگترین ضلع T1، برابر m باشد، در اینصورت هر کدام از اضلاع T2 به طور جداگانه، میتوانند در بازه 0 تا m تغییر کنند، اما نه به طور همزمان. بزرگترین مقدار برای هر ضلع را چه در نظر می گیرید؟ توجه دارید که اگر x و y دو متغیر مستقل باشند، آنگاه z=x+y زمانی بیشینه می شود که هر کدام از x و y بیشینه شوند. اما اگر x و y به یکدیگر وابسته باشند، نمیتوان چنین گفت، و شاید امکان نداشته باشد هر دو همزمان بیشینه شوند.


1 - برای ساده تر شدن مساله یکی از محور ها را (به نام x) منطبق (حالت خاصی از توازی) بر یکی از اضلاع مثلث T1 در نظر می گیریم . هم چنین باز هم برای ساده شدن ، مبدا مختصات را نیز منطبق بر یکی از دو راس سر ضلع مذکور فرض می کنیم.
اکنون بزرگ ترین ضلعی که از نظر مختصه ی اول ممکن است درون این مثلث جای گیرد ، ضلعی است موازی ضلع منطبق بر محور x ها و با اختلاف ناچیز از آن . در هر حال این ضلع ، کوچک تر از ضلع منطبق بر محور x هاست.

منظور شما از "بزرگترین ضلع از نظر مختصه اول" چیست؟

Kesel
11-07-2012, 11:56
این گزاره کمی ابهام دارد. اگر طول بزرگترین ضلع T1، برابر m باشد، در اینصورت هر کدام از اضلاع T2 به طور جداگانه، میتوانند در بازه 0 تا m تغییر کنند، اما نه به طور همزمان. بزرگترین مقدار برای هر ضلع را چه در نظر می گیرید؟ توجه دارید که اگر x و y دو متغیر مستقل باشند، آنگاه z=x+y زمانی بیشینه می شود که هر کدام از x و y بیشینه شوند. اما اگر x و y به یکدیگر وابسته باشند، نمیتوان چنین گفت، و شاید امکان نداشته باشد هر دو همزمان بیشینه شوند.



درست می فرمایید.شاید باید از کلمه ی ممکن استفاده می کردم.درهر حال منظور از «بزرگترین مقدار خود» ، مقداری است که بیش تر از آن ممکن نباشد . طبیعتا برای رسم ضلع اول محدودیتی نداریم اما برای رسم اضلاع دیگر قاعدتا عدم استقلال ، محدودیتی ایجاد می کند . این محدودیت در جواب کلی مساله تاثیر گذار نیست و صرفا بزرگترین مقدار ممکن مد نظر است.

ما باید محور های مختصات را طوری قرار دهیم که بزرگترین ضلع قابل رسم درون مثلث T1 ، دقیقا همان ضلعی باشد که بیشترین مختصه ی اول را دارد. یعنی اگر مبدا مختصات را طوری انتخاب کردیم که بزرگترین پاره خط درون مثلث ، دارای بزرگترین مختصه ی اول نبود ، فورا مبدا را روی یکی از دو راس دیگر تنظیم می کنیم تا بالاخره این شرط برقرار شود.به صورت خیلی ساده تر محور x ها را منطبق بر بزرگترین ضلع می گیریم ولی چون به کار بردن وسیله ی اندازه گیری شاید قابلیت استدلال نداشته باشد از عبارت «منطبق بر بزرگترین ضلع» پرهیز کردم.
به این اشکال توجه کنید :
[ برای مشاهده لینک ، لطفا با نام کاربری خود وارد شوید یا ثبت نام کنید ]

اما برای رسم ضلع دوم ، محدودیت این است که این ضلع حتما باید از یکی از دو سر ضلعی که در مرحله ی 1 رسم کردیم ، رسم شود. توجه کنید که مجموع دو ضلع دوم و سوم زمانی حد اکثر می شود که دقیقا چسبیده به اضلاع مثلث T1 باشند.(بنا به قضیه ی *)

قضیه ی *

شرط : اگر نقطه ی K درون مثلث ABC باشد.

حکم : BK+KC<AB+AC

موارد استفاده :

با استفاده از این قضیه می توان اثبات کرد:
الف : که مجموع دو ضلع مثلث T2 درصورتی بیشینه است که بی نهایت به دو ضلع مثلث T1 نزدیک باشد.
ب : در هر صورت مجموع دو ضلع مثلث درونی کوچک تر از مجموع دو ضلع مثلث بیرونی است.

توضیحات : چنین قضیه ای در هندسه به چشم من نخورده است.برای همین ممکن است این قضیه قبلا مطرح و یا حتی اثبات شده باشد.در هر حال من در این مساله نیازمند طرح این قضیه بودم.اگر جایی مطرح شده یا اثبات دیگری یا مشابه دارد ، این اثبات را هم به آن ها اضافه کنید اگر که نه ، گویا باید به نام من شود (!):20:

شکل :
[ برای مشاهده لینک ، لطفا با نام کاربری خود وارد شوید یا ثبت نام کنید ]

متن اثبات قضیه :

ازنقطه ی A خطی عمود بر ضلع BC رسم می کنیم . نقطه ی D را طوری انتخاب می کنیم که اولا D روی AH قرار گیرد و ثانیا :

رابطه ی 1 : [ برای مشاهده لینک ، لطفا با نام کاربری خود وارد شوید یا ثبت نام کنید ]

توجه داریم که :

[ برای مشاهده لینک ، لطفا با نام کاربری خود وارد شوید یا ثبت نام کنید ](x)=\frac{BH}{BD}&space;,&space;cos(y)=\frac{BH}{ AB}

لذا خواهیم داشت :

[ برای مشاهده لینک ، لطفا با نام کاربری خود وارد شوید یا ثبت نام کنید ]{x}<\hat{y}\Rightarrow&space;cos(x)>cos(y)\Rightarrow&space;\frac{BH}{BD}>\frac{BH}{AB}\Rightarrow&space;AB>BD

به طور مشابه می توان اثبات کرد :

[ برای مشاهده لینک ، لطفا با نام کاربری خود وارد شوید یا ثبت نام کنید ]>DC

با جمع زدن طرفین دو نامساوی بالا داریم :

[ برای مشاهده لینک ، لطفا با نام کاربری خود وارد شوید یا ثبت نام کنید ]>BD&plus;DC

با توجه به شرط (رابطه ی 1) و چایگذاری در سمت راست عبارت خط قبل حکم ثابت می شود :

[ برای مشاهده لینک ، لطفا با نام کاربری خود وارد شوید یا ثبت نام کنید ]>BK&plus;KC


همانطور که دیدیم در این قضیه ، ضلع اول هر دو مثلث را منطبق گرفتیم . بنابراین اگر حکم برای حالت انطباق درست باشد ، برای حالتی که ضلع مثلث درونی فاصله ای ناچیز از بیرونی دارد نیز صادق خواهد بود و مجموع سه ضلع T2 حتی کوچکتر از مقدار به دست آمده در قضیه خواهد شد. (قیاس اولویت)

ضمنا استفاده از قضیه ی فیثاغورس فقط در حالت خاصّ وجود زوایه ی قائمه قابل استدلال است و مطرح کردن این قضیه در این مساله در حالت کلی چندان منطقی به نظر نمی رسد.

پ.ن : اثبات قضیه ی بالا با استفاده از قضایای دیگر هم امکان پذیر است مثل «ضلع رو به رو به زاویه ی بزرگتر ، بزرگتر از ضلع رو به رو به زاویه ی کوچکتر است» منتهی اثبات کوچکتر یا بزرگتر بودن زاویه ، کار را دشوار می کند

ممنون از توجه شما

Headphone
14-07-2012, 21:37
سلام

میدونیم که حجم قطاعی از یک کره ( برش وتری ) رو از فرمول [ برای مشاهده لینک ، لطفا با نام کاربری خود وارد شوید یا ثبت نام کنید ] بدست میارن . که R شعاع کره و h ارتفاع قطاع هستش . ( البته اگه درست خاطرم باشه )

کنجکاو شدم ببینم که آیا اثباتی برای این فرمول وجود داره ؟ ( حتما وجود داره [ برای مشاهده لینک ، لطفا با نام کاربری خود وارد شوید یا ثبت نام کنید ] )

lebesgue
17-07-2012, 01:07
[ برای مشاهده لینک ، لطفا با نام کاربری خود وارد شوید یا ثبت نام کنید ]



اگر از روش دیسک ([ برای مشاهده لینک ، لطفا با نام کاربری خود وارد شوید یا ثبت نام کنید ]) استفاده کنید، میتوانید ببینید که حجم مورد نظر برابر با انتگرال زیر است:



[ برای مشاهده لینک ، لطفا با نام کاربری خود وارد شوید یا ثبت نام کنید ]{R-h}^{R}(R^2-x^2)dx


که پس از محاسبه و ساده سازی، به همان عبارت شما میرسد.

lebesgue
18-07-2012, 11:39
Kesel گرامی،

از آنجا که پرسش، اثبات یک گزاره نسبتاً بدیهی را میخواهد، انتظار داریم گزاره ای نابدیهی تر از آن، برای اثبات بکار گرفته نشود. این درست است که اگر طول بزرگترین ضلع T1 برابر m باشد، امکان ندارد که هیچ کدام از اضلاع T2 بزرگتر از m باشند، اما در جای خود، نیاز به اثبات دارد. آن قضیه ای هم که بیان کردید، در صورتی قابل استفاده است که ابتدا نشان دهید حتماً یکی از اصلاع T2 باید منطبق بر یک ضلع T1 باشد. در ضمن این را اضافه کنم که، اگر چه در صورت سوال بطور روشن اشاره نشده، اما اضلاع و رئوس مثلث T2 میتوانند روی اضلاع مثلث T1 باشند.

البته ما در اینجا به دنبال ساده ترین اثباتها هستیم، اما اثبات شما با اضافه کردن چند گام میانی، به نظر من درست است. من نیز چند اثبات در نظر داشتم، که شاید در فرصتی دیگر آنها را بنویسم.

-------------------------------
پرسشی که مطرح شد، در واقع پیش درآمدی بود برای پرسش زیر:

چهاروجهی T2 درون چهاروجهی T1 جای دارد. آیا امکان دارد که مجموع طول شش یال T2 بزرگتر از مجموع طول شش یال T1 باشد؟ درستی پاسخ خود را نشان دهید.

منبع: اضافه خواهد شد.

Kesel
21-07-2012, 18:13
متشکرم
اگر امکان داره قبل از حل و رفع پرسش چهار وجهی ، مساله ی مثلث رو به طور کامل پاسخ دهید چون اولا پیش درآمدی بر این پرسش دوم هست و ثانیا مفاهیمی مثل "بدیهی تر" یا "نابدیهی تر" متر و مقیاس دقیقی ندارند.اگر اثبات دیگه ای هست من خوشحال می شم بدونمش. این مساله رو جنابعالی توی هوپا هم مطرح فرمودید منتهی باز هم جوابی پذیرفته نشده واسه همین خیلی مشتاقم راه اثباتش رو بدونم.یا اگر فکر می کنید می شه بین گام های راه حل من گام دیگه ای گذاشت باز هم خوشحال می شم بدونمش.
با تشکر

Kesel
26-07-2012, 17:32
لطفا حذف شود

Kesel
10-08-2012, 11:44
من بیشتر رفتم روی این مساله فکر کردم و یه اثبات دیگه براش نوشتم . اولش یه نقص هایی داشت که الان فرصت کردم برطرفشون کردم . به نظرم این راه حل اصلا قسمت مبهمی نداشته باشه.
خوشحال می شم نظرتون رو بدونم .
مطابق شکل یک دایره درون هر مثلث محاط می کنیم.
[ برای مشاهده لینک ، لطفا با نام کاربری خود وارد شوید یا ثبت نام کنید ]
چون هر دایره درون مثلث محاط شده است بنابراین مساحت هر دایره کوچک تر از مساحت مثلث محیط خود است (طبق تعریف محاط بودن) :

[ برای مشاهده لینک ، لطفا با نام کاربری خود وارد شوید یا ثبت نام کنید ]^{2}<&space;\frac{ah}{2}\Rightarrow&space;r^{2}<ah
[ برای مشاهده لینک ، لطفا با نام کاربری خود وارد شوید یا ثبت نام کنید ]^{2}<&space;\frac{a'h'}{2}\Rightarrow&space;R^{2}<a'h'

با تقسیم طرفین دو عبارت بالا خواهیم داشت:

[ برای مشاهده لینک ، لطفا با نام کاربری خود وارد شوید یا ثبت نام کنید ]{\color{DarkGreen}&space;\Rightarrow&space;(\frac{r} {R})^{2}<\frac{ah}{a'h'}}

همچنین می دانیم مساحت هر یک از مثلث های T1 ویا T2 برابر با مجموع مساحت های سه مثلث به وجود آمده در شکل است :

[ برای مشاهده لینک ، لطفا با نام کاربری خود وارد شوید یا ثبت نام کنید ]{T1}=\frac{ra}{2}&plus;\frac{rb}{2}&plus;\frac{r c}{2}=\frac{ah}{2}\Rightarrow&space;a&plus;b&plus;c=\frac{ah}{r}

[ برای مشاهده لینک ، لطفا با نام کاربری خود وارد شوید یا ثبت نام کنید ]{T2}=\frac{Ra'}{2}&plus;\frac{Rb'}{2}&plus;\frac {Rc'}{2}=\frac{a'h'}{2}\Rightarrow&space;a'&plus;b'&plus;c'=\frac{ a'h'}{R}

اکنون به بیان و اثبات قضیه ی زیر می پردازیم :
قضیه : اگر مثلث T2 درون مثلث T1 باشد ، آنگاه محیط T1 بزرگتر از محیط T2 خواهد بود.

اثبات :
برهان خلف : فرض می کنیم که محیط T1 کوچک تر یا مساوی محیط T2 است :

[ برای مشاهده لینک ، لطفا با نام کاربری خود وارد شوید یا ثبت نام کنید ]'&plus;b'&plus;c'\Rightarrow&space;\frac{ah}{ r}\leq&space;\frac{a'h'}{R}\Rightarrow&space;{\color{DarkGreen }&space;\frac{ah}{a'h'}\leq&space;\frac{r}{R}}

با توجه به روابط سبز رنگ نتیجه می گیریم :

[ برای مشاهده لینک ، لطفا با نام کاربری خود وارد شوید یا ثبت نام کنید ]{\color{DarkGreen}&space;(\frac{r}{R})^{2}<\frac{ah}{a'h'}\leq&space;\frac{r}{R}}\Rightarrow&space;(\frac {r}{R})^{2}<\frac{r}{R}\Rightarrow&space;{\color{Red}&space;R>r}

اما در صورت قضیه فرض شده که مثلث T2 درون مثلث T1 است ، یعنی مساحت T1 بزرگتر از مساحت T2 می باشد :

[ برای مشاهده لینک ، لطفا با نام کاربری خود وارد شوید یا ثبت نام کنید ]{T1}>S_{T2}\Rightarrow&space;\frac{ah}{2}>\frac{a'h'}{2}\Rightarrow&space;ah>a'h'

بنا به دومین رابطه ی سبز داریم :

[ برای مشاهده لینک ، لطفا با نام کاربری خود وارد شوید یا ثبت نام کنید ]{\color{DarkGreen}&space;\frac{ah}{a'h'}\leq&space;\ frac{r}{R}}\xrightarrow[]{ah>a'h'}{\color{Red}&space;r\geq&space;R}

که این مغیر با نتیجه ی به دست آمده ای است که در بالا با رنگ قرمز مشخص شده است ، لذا در مسیر اثبات نقیض حکم به تناقض منطقی برخورد کردیم ؛ بنابراین نقیض حکم باطل و حکم اثبات می شود.

ali_hp
10-08-2012, 15:06
درست می فرمایید.شاید باید از کلمه ی ممکن استفاده می کردم.درهر حال منظور از «بزرگترین مقدار خود» ، مقداری است که بیش تر از آن ممکن نباشد . طبیعتا برای رسم ضلع اول محدودیتی نداریم اما برای رسم اضلاع دیگر قاعدتا عدم استقلال ، محدودیتی ایجاد می کند . این محدودیت در جواب کلی مساله تاثیر گذار نیست و صرفا بزرگترین مقدار ممکن مد نظر است.

ما باید محور های مختصات را طوری قرار دهیم که بزرگترین ضلع قابل رسم درون مثلث T1 ، دقیقا همان ضلعی باشد که بیشترین مختصه ی اول را دارد. یعنی اگر مبدا مختصات را طوری انتخاب کردیم که بزرگترین پاره خط درون مثلث ، دارای بزرگترین مختصه ی اول نبود ، فورا مبدا را روی یکی از دو راس دیگر تنظیم می کنیم تا بالاخره این شرط برقرار شود.به صورت خیلی ساده تر محور x ها را منطبق بر بزرگترین ضلع می گیریم ولی چون به کار بردن وسیله ی اندازه گیری شاید قابلیت استدلال نداشته باشد از عبارت «منطبق بر بزرگترین ضلع» پرهیز کردم.
به این اشکال توجه کنید :
[ برای مشاهده لینک ، لطفا با نام کاربری خود وارد شوید یا ثبت نام کنید ]

اما برای رسم ضلع دوم ، محدودیت این است که این ضلع حتما باید از یکی از دو سر ضلعی که در مرحله ی 1 رسم کردیم ، رسم شود. توجه کنید که مجموع دو ضلع دوم و سوم زمانی حد اکثر می شود که دقیقا چسبیده به اضلاع مثلث T1 باشند.(بنا به قضیه ی *)

قضیه ی *

شرط : اگر نقطه ی K درون مثلث ABC باشد.

حکم : BK+KC<AB+AC

موارد استفاده :

با استفاده از این قضیه می توان اثبات کرد:
الف : که مجموع دو ضلع مثلث T2 درصورتی بیشینه است که بی نهایت به دو ضلع مثلث T1 نزدیک باشد.
ب : در هر صورت مجموع دو ضلع مثلث درونی کوچک تر از مجموع دو ضلع مثلث بیرونی است.

توضیحات : چنین قضیه ای در هندسه به چشم من نخورده است.برای همین ممکن است این قضیه قبلا مطرح و یا حتی اثبات شده باشد.در هر حال من در این مساله نیازمند طرح این قضیه بودم.اگر جایی مطرح شده یا اثبات دیگری یا مشابه دارد ، این اثبات را هم به آن ها اضافه کنید اگر که نه ، گویا باید به نام من شود (!):20:

شکل :
[ برای مشاهده لینک ، لطفا با نام کاربری خود وارد شوید یا ثبت نام کنید ]

متن اثبات قضیه :

ازنقطه ی A خطی عمود بر ضلع BC رسم می کنیم . نقطه ی D را طوری انتخاب می کنیم که اولا D روی AH قرار گیرد و ثانیا :

رابطه ی 1 : [ برای مشاهده لینک ، لطفا با نام کاربری خود وارد شوید یا ثبت نام کنید ]

توجه داریم که :

[ برای مشاهده لینک ، لطفا با نام کاربری خود وارد شوید یا ثبت نام کنید ](null)

لذا خواهیم داشت :

[ برای مشاهده لینک ، لطفا با نام کاربری خود وارد شوید یا ثبت نام کنید ](null)

به طور مشابه می توان اثبات کرد :

[ برای مشاهده لینک ، لطفا با نام کاربری خود وارد شوید یا ثبت نام کنید ](null)

با جمع زدن طرفین دو نامساوی بالا داریم :

[ برای مشاهده لینک ، لطفا با نام کاربری خود وارد شوید یا ثبت نام کنید ](null)

با توجه به شرط (رابطه ی 1) و چایگذاری در سمت راست عبارت خط قبل حکم ثابت می شود :

[ برای مشاهده لینک ، لطفا با نام کاربری خود وارد شوید یا ثبت نام کنید ](null)


همانطور که دیدیم در این قضیه ، ضلع اول هر دو مثلث را منطبق گرفتیم . بنابراین اگر حکم برای حالت انطباق درست باشد ، برای حالتی که ضلع مثلث درونی فاصله ای ناچیز از بیرونی دارد نیز صادق خواهد بود و مجموع سه ضلع T2 حتی کوچکتر از مقدار به دست آمده در قضیه خواهد شد. (قیاس اولویت)

ضمنا استفاده از قضیه ی فیثاغورس فقط در حالت خاصّ وجود زوایه ی قائمه قابل استدلال است و مطرح کردن این قضیه در این مساله در حالت کلی چندان منطقی به نظر نمی رسد.

پ.ن : اثبات قضیه ی بالا با استفاده از قضایای دیگر هم امکان پذیر است مثل «ضلع رو به رو به زاویه ی بزرگتر ، بزرگتر از ضلع رو به رو به زاویه ی کوچکتر است» منتهی اثبات کوچکتر یا بزرگتر بودن زاویه ، کار را دشوار می کند

ممنون از توجه شما
ظا هرا شما در ابتداي اثبات ، از اين استفاده مي كني كه هر پاره خطي داخل مثلث از بزرگترين ضلع مثلث كوچكتر است. بعد ميخواي از قضيه اي كه مطرح كردي نتيجه بگيري كه كه مجموع دو پاره خط ديگه هم از دو ضلع ديگه مثلث كو چكتره .كه البته اين قسمت به نظر من ابهام داره ، حتي جمله " دقيقا چسبيده به اضلاع مثلث باشن " معناي رياضيش مشخص نيست .
و البته مشخص نيست كه بايد چسبيده به كدام دو ضلع باشن! اگه چسبيده به ضلع بزرگتر و يه ضلعه ديگه باشن مشكل ايجاد ميشه در ادامه راه حل.
البته اثبات قضيه تون هم درست نيست ، چون وجود نقطه D با خواصي كه شما مد نظرتون هست بديهي نيست . و اصلا يه جورايي معادل حكم قضيه است !( البته اثبات وجود نقطه D از خود قضيه سخت تره به نظره من)
البته قضيه با ايده اي مشا به ايده شما قابل حله ، و بعد هم با همين قضيه ميشه مساله رو حل كرد.
يه نكته ساده كه ميتونه مفيد باشه ، اينه كه در يك مثلث ضلع رو برو به زاويه منفرجه بزرگترين ضلع هست.
راستي بابت قضيه اي هم كه كشف كردين تبريك ، به هر حال چه جديد باشه چه نباشه ( كه نيست) ، به نام شما هم مي شود!
در مورد اين راه حل اخرتونم ، همون اولش دو تا نامساوي همجهت رو به هم تقسيم كردين ! كه مشكل داره اين كار!
به نظر من روند اولتون كه سعي ميكردين چيزايي كه درك و شهودتون بهتون ميگه رو به زبون رياضي بنويسين ، بهتر از اينه كه بخواين با بازي با فرمولها مساله رو حل كنين.
احتمالا شما هم حس مي كني كه اين مساله خيلي سادست و حكمش اصلان قوي نيست! ولي مشكل اينه كه خيلي هم وحشيه! يعني يه مثلث دلخواه كه داخل مثلث ديگه محاطه شايد يه جورايي ارتباط كمي با مثلث اصلي داره ، پيشنهاد من اينه كه سعي كنيد با دنباله اي از تغييرات كه محيط مثلث رو كم نميكنن ، اين مثلث وحشي دروني رو به يك مثلث اهلي تر تبديل كنيد! كه ارتباطات بهتري با مثلث بيروني داره.

Kesel
10-08-2012, 18:07
دوست من ای کاش نقد راه حل قبلی رو یک ماه پیش می نوشتید تا من همونجا ازش استفاده می کردم.نمی تونم درک کنم چی باعث شده بعد از یک ماه پاسخ بدید . در هر حال ممنون


البته قضيه با ايده اي مشا به ايده شما قابل حله ، و بعد هم با همين قضيه ميشه مساله رو حل كرد.

چرا پس حلش نمی کنید ؟ من اینو توی پست قبلی ازتون درخواست کردم


راستي بابت قضيه اي هم كه كشف كردين تبريك ، به هر حال چه جديد باشه چه نباشه ( كه نيست) ، به نام شما هم مي شود!

اصلا منطورتون رو متوجه نمی شم . من وقتی دارم بین صحبت هام شوخی می کنم شما باید مسخره کنید ؟ نمی تونستید فقط جایی که این قضیه اثبات شده رو ذکر کنید ؟ هیچ معنایی غیر از تمسخر از این جملتون نمی بینم


در مورد اين راه حل اخرتونم ، همون اولش دو تا نامساوي همجهت رو به هم تقسيم كردين ! كه مشكل داره اين كار!

دقیقا ! این چیزیه که به عنوان کمک از شما قبول می کنم . کاملا به جا اشاره فرمودید و راهکار من اشتباه هست (اگر ماه بعد کسی این رو تکرار نکنه)


به نظر من روند اولتون كه سعي ميكردين چيزايي كه درك و شهودتون بهتون ميگه رو به زبون رياضي بنويسين ، بهتر از اينه كه بخواين با بازي با فرمولها مساله رو حل كنين.

شما اگه می تونید اونو ادامه بدید من خوشحال می شم . من حدود یک ماهه که دارم روش فکر می کنم و به این نتیجه رسیدم که بهترین راه برهان خلفه چون به قول شما مثلث T2 خیلی وحشیه . البته نمی گم راه دیگه ای نیست . چرا هست ولی لطفا ارائش کنید .


احتمالا شما هم حس مي كني كه اين مساله خيلي سادست و حكمش اصلان قوي نيست!

نه اصلا هم فکر نمی کنم ساده هست . کسی که ده دقیقه با این مساله کار کنه متوجه می شه که ساده نیست چه برسه به من که نصف یه دفتر 200 برگ رو به خاطر همین یک مساله سیاه کردم.


پيشنهاد من اينه كه سعي كنيد با دنباله اي از تغييرات كه محيط مثلث رو كم نميكنن ، اين مثلث وحشي دروني رو به يك مثلث اهلي تر تبديل كنيد!

می شه بیشتر توضیح بدید؟
با تشکر

ali_hp
10-08-2012, 22:09
راستش من نقد راه حل قبليرو همون يك ماه پيش كامل تايپ كرده بودم ، ولي ارسال نشد ، و بعدش هم فراموش كردم كه ارسالش كنم!
دوست عزيز ، من تمسخر نكردم! واقعا تبريك گفتم! همونطوري كه وقتي خودم چيز جديديرو مي فهمم به خودم تبريك ميگم!من واقعا هدفم تحسين روند فكري شما بود فقط!نه چيز ديگه اي.
براي اثبات قضيه تون كافي است عمودي از K بر BC رسم كنيد ، و محل برخورد امتداد اين عمود با AC را D بناميد ، حال با توجه به منفرجه بودن زواياي BKD و CKD و يا همانطور كه خودتان مشابها ثابت كرديد مي دانيم كه
BK+CK < BD + CD
حال با توجه به نامساوي مثلث ، داريم :
BD < BA + DA
كه با جمع زدن اين دو نامساوي حكم قضيه نتيجه ميشه .
دقت كنيد كه در اين راه حل هم ما با دنباله اي از تغييرات مثلث KBC رو اهلي تر كرديم!
حالا برگرديم به مساله اصلي :
اول اينكه منظور من از سادگي اين بود لحظه اولي كه اين سوالو ميبينيم فكر مي كنيم حكمش چيز واضحييه و ما حس مي كنيم كه مساله سادست .
مشابه ايده اثبات قضيه عمل مي كنيم!
هرچند ظاهرا در مورد مثلثهاي محاط در يك مثلث ، اونايي كه راسهاشون روي اضلاع مثلث بيروني هست ميتونن محيطهاي بيشتري داشته باشن ، اما مساله در مورداينها ساده تره ، چون اينا مثلثهايي اهلي ترن!
در مورد مثلثهايي كه راسهاشون روي ضلعهاي مثلث اصلي هست ، كافيه سه بار نامساوي مثلث نوشته و با هم جمع كنيم تا حكم ثابت شه.
در مورد يك مثلث دلخواه داخل مثلث بيروني ، هدف اينه كه سعي كنيم با يه تغييراتي كه محيطو كم نمي كنن راس هاشو بندازيم روي اضلاع ( مثل كاري كه در مورد قضيه كرديم!)
مثلث داخلي رو xyz مي ناميم ارتفاع xh رو رسم كنيد تا امتدادش از طرف x محيط مثلث در نقطه q قطع كنه ، حال طبق قضيه ( البته اگه xyz زواياش حاده باشه) qyz محيطش بيشتر از xyz است و با ادامه روند مي توان همه راسها رو روي ضلعها انداخت!
در حالتي هم كه بعضي از مثلثهاي اين روند زاويه منفرجه داشته باشن به سادگي قابل اصلاحه.
البته از همون اول هم ميشه يه جوري مثلث داخلي xyz رو به مثلث بيروني به طور خوبي ربط داد، كافيه ضلع xy رو از طرف y و ضلع yz رو از طرف z و ضلع zx رو از طرف x امتداد بديم تا اضلاع مثلث ABC رو قطع كنن ، حالا با نوشتن چند تا نامساوي مناسب !و جمع كردن اونها ميشه حكمو نتيجه گرفت!

Kesel
11-08-2012, 15:42
بسیار هوشمندانه بود ! به نظر خودم تنها نقطه ی تاریک اثبات قضیه ی * اثبات وجود نقطه ی D بود که به قول شما اثباتش به دشواری اثبات خود حکم هست . ایده ی استفاده از قضیه ی حمار برای اثبات قضیه ی * خیلی محدودیت ها رو به نظرم کنار می گذاره .
راه حلی که شما فرمودید استفاده ی چندین و چند باره از این قضیه ی * هست که جالب بود و تونسته بودید مثلث T2 رو اهلی کنید.
یعنی هر سه راس مثلث T2 رو انداختید روی اضلاع T1 .
حالا که از نامساوی مثلثی برای اثبات قضیه ی * استفاده کردید راه زیر به نظرم ساده تر از چند بار استفاده کردن از قضیه ی * و در نهایت استفاده ی مجدد از قضیه ی حمار هست و این که برهان خلف هم داره آخرش (برهان مورد علاقه ی من!) :

می دونیم که هم دوران و هم انتقال ایزومتری هستند . یعنی فاصله ی نقاط رو تغییر نمی دن و در نتیجه محیط تغییری نمی کنه . یکی از رئوس غیر منفرجه ی مثلث T2 رو انتقال می دیم روی یکی از اضلاع T1 (مهم نیست کدوم ضلع). حالا حول همون راس ، مثلث T2 رو دوران می دیم تا ضلع رو به رو به زاویه ی منفرجه یا قائمه (اگر زاویه ی منفرجه یا قائمه داشتیم در غیر این صورت ضلع رو به رو به هر زاویه ای) ، منطبق بر ضلع مثلث T1 بشه . و به شکل زیر از از قضیه ی * برای اثبات حکم استفاده می کنیم :


[ برای مشاهده لینک ، لطفا با نام کاربری خود وارد شوید یا ثبت نام کنید ]


مثلث ABC همان مثلث T1 است و DEF همان دوران و انتقال یافته ی مثلث T2 .
خط HJ عمود بر ضلع BC و قطع کننده ی ضلع AC در نقطه ی J است. نقطه ی J را به دو نقطه B و C وصل می کنیم (B و C نقاط ابتدا و انتهای ضلعی هستند که در قسمتی از این ضلع مثلث ها با هم مشترکند)
با استفاده از قضیه ی * داریم :

[ برای مشاهده لینک ، لطفا با نام کاربری خود وارد شوید یا ثبت نام کنید ]{\color{Red}&space;DJ&plus;FJ>DE&plus;EF}

قبلا شرط کردم که زوایای D و F حق منفرجه یا قائمه بودن را ندارند در غیر این صورت دوران داده می شدند تا جای نقطه ی کنونی E رو پر کنند .

زوایای BDJ و CFJ منفرجه هستند .(چون EFH و HDE حاده هستند (چون ارتفاع EH درون مثلث است)) بنابراین : (در هر مثلث ضلع رو به رو به زاویه ی منفرجه بزرگترین ضلع است)

[ برای مشاهده لینک ، لطفا با نام کاربری خود وارد شوید یا ثبت نام کنید ]>DJ&space;,&space;CJ>FJ

با جمع زدن دو نامساوی بالا داریم :

[ برای مشاهده لینک ، لطفا با نام کاربری خود وارد شوید یا ثبت نام کنید ]{DarkGreen}&space;BJ&plus;CJ%3EDJ&plus;FJ

که بنا بر قضیه ی * نتیجه می شود :

[ برای مشاهده لینک ، لطفا با نام کاربری خود وارد شوید یا ثبت نام کنید ]{\color{DarkGreen}&space;AB&plus;AC>BJ&plus;CJ}

با با جمع زدن دو نامساوی سبز رنگ بالا ثابت می شود :

[ برای مشاهده لینک ، لطفا با نام کاربری خود وارد شوید یا ثبت نام کنید ]

حال این نامساوی را با نامساوی قرمز رنگ بالا جمع می زنیم تا به دست بیاید :

[ برای مشاهده لینک ، لطفا با نام کاربری خود وارد شوید یا ثبت نام کنید ]>DE&plus;EF

حالا با توجه به عبارت بالا اگر قرار باشد محیط مثلث T2 بزرگتر از محیط مثلث T1 باشد ، ایجاب می کند که ضلع DF بزرگتر از ضلع BC باشد (به سادگی می توان نشان داد) که این با فرض در تناقض است . ما فرض کردیم مثلث T2 درون مثلث T1 باشد که موجب این است که هیچ یک از اضلاع T2 از T1 خارج نشود .



البته از همون اول هم ميشه يه جوري مثلث داخلي xyz رو به مثلث بيروني به طور خوبي ربط داد، كافيه ضلع xy رو از طرف y و ضلع yz رو از طرف z و ضلع zx رو از طرف x امتداد بديم تا اضلاع مثلث ABC رو قطع كنن ، حالا با نوشتن چند تا نامساوي مناسب !و جمع كردن اونها ميشه حكمو نتيجه گرفت!


این قسمت رو متوجه نشدم.یعنی می گید محل برخورد رئوس T2 و اضلاع T1 رو به هم وصل کنیم ؟ از کجا معلوم محیط این مثلث جدید بیشتر از T2 باشه؟ نامساوی مثلث رو برای چه چیزی می نویسید ؟

ali_hp
12-08-2012, 01:38
انتقال و دوران ایده جالبیه ، فقط باید بگین که چرا میشه با انتقال و دوران طوری دو راس مثلث داخلی رو روی یک ضلع مثلث بیرونی انداخت که " تمام مثلث داخلی" همچنان داخل مثلث بیرو نی باقی بمونه!
در مورد اون راه حل دیگه هم:
54302

Kesel
12-08-2012, 14:30
انتقال و دوران ایده جالبیه ، فقط باید بگین که چرا میشه با انتقال و دوران طوری دو راس مثلث داخلی رو روی یک ضلع مثلث بیرونی انداخت که " تمام مثلث داخلی" همچنان داخل مثلث بیرو نی باقی بمونه!
در مورد اون راه حل دیگه هم:
54302

اگر داخل هم نیفته باز هم مشکلی ایجاد نمی کنه :
فرض می کنیم در هیچ صورتی ممکن نباشد که با دوران و انتقال ، مثلث T2 کاملا درون T1 قرار گیرد لذا ایجاب می کند :

[ برای مشاهده لینک ، لطفا با نام کاربری خود وارد شوید یا ثبت نام کنید ]'_{1}>h_{1},h'_{2}>h_{2},h'_{3}>h_{3}

چون مثلث T2 درون T1 واقع شده ، لذا مساحت کمتری دارد :

[ برای مشاهده لینک ، لطفا با نام کاربری خود وارد شوید یا ثبت نام کنید ]{T2}<S_{T1}\Rightarrow&space;a'h'_{1}<ah_{1}\Rightarrow&space;\frac{h_{1}}{h'_{1}}>\frac{a'}{a}\xrightarrow[]{h'_{1}>h_{1}}{\color{DarkGreen}&space;a>a'}
[ برای مشاهده لینک ، لطفا با نام کاربری خود وارد شوید یا ثبت نام کنید ]{T2}<S_{T1}\Rightarrow&space;b'h'_{2}<bh_{2}\Rightarrow&space;\frac{h_{2}}{h'_{2}}>\frac{b'}{b}\xrightarrow[]{h'_{2}>h_{2}}{\color{DarkGreen}&space;b>b'}
[ برای مشاهده لینک ، لطفا با نام کاربری خود وارد شوید یا ثبت نام کنید ]{T2}<S_{T1}\Rightarrow&space;c'h'_{3}<ch_{3}\Rightarrow&space;\frac{h_{3}}{h'_{3}}>\frac{c'}{c}\xrightarrow[]{h'_{3}>h_{3}}{\color{DarkGreen}&space;c>c'}

با جمع زدن سه نامساوی سبز حکم اثبات می شه و دیگه نیازی هم به قضیه * و حمار نخواهد بود.
ضمنا این راه حلی که توضیح دادید هم کوتاهه هم خیلی شیرینه.به نظرم از این راه حل شما ساده تر نداشته باشه این مساله.
با تشکر

ali_hp
13-08-2012, 01:06
اگر داخل هم نیفته باز هم مشکلی ایجاد نمی کنه :
فرض می کنیم در هیچ صورتی ممکن نباشد که با دوران و انتقال ، مثلث T2 کاملا درون T1 قرار گیرد لذا ایجاب می کند :

[ برای مشاهده لینک ، لطفا با نام کاربری خود وارد شوید یا ثبت نام کنید ]'_{1}>h_{1},h'_{2}>h_{2},h'_{3}>h_{3}

اين قسمت استدلال كامل نيست ، دقيقا چرا ايجاب مي كنه؟
دقت كنيد كه براي اينكه بتوانيم مثلث داخلي رو به شكلي كه شما مد نظرتونه در بياريم ، مشكل فقط بزرگ بودن انداز ارتفاع مثلث داخلي نيست!

Kesel
13-08-2012, 13:37
اين قسمت استدلال كامل نيست ، دقيقا چرا ايجاب مي كنه؟
دقت كنيد كه براي اينكه بتوانيم مثلث داخلي رو به شكلي كه شما مد نظرتونه در بياريم ، مشكل فقط بزرگ بودن انداز ارتفاع مثلث داخلي نيست!

واضحه . شما اگر محور طول ها رو روی ضلع CB در نظر بگیرید ، برای این که مثلث T2 بیرون از T1 باشه لازمه که نقطه ای در مثلث T2 وجود داشته باشه که عرض بیشتری از عریض ترین نقطه ی T1 داشته باشه. ضمن این که توجه کنید که هر سه ارتفاع T2 باید بزرگتر از هر سه ارتفاع T1 باشد . در صورتی که فقط دو ارتفاع یا فقط یک ارتفاع بلند تر داشته باشیم با دوران می تونیم T2 رو درون T1 جا بدیم (چون یک یا دو ارتفاعش کوچک تر از T1 خواهد شد)


دقت كنيد كه براي اينكه بتوانيم مثلث داخلي رو به شكلي كه شما مد نظرتونه در بياريم ، مشكل فقط بزرگ بودن انداز ارتفاع مثلث داخلي نيست!

چه مشکل دیگه ای می تونه وجود داشته باشه ؟

ali_hp
13-08-2012, 16:38
واضح نيست ! باز هم اثباتي از ادعاتون ارايه نكردين!خطرناكترين كلمه رياضيات " واضح " هست!
البته از نظر شهودي هم حرفتون واضح نيست ، ممكنه نقاطي از مثلث داخلي بدون اينكه ارتفاع بيشتري از ارتفاع مثلث بيروني داشته باشن ، خارج از مثلث بيروني قرار بگيرن ، بخصوص وقتي قاعده مثلث داخلي خيلي جاي مانور نداشته باشه و یکی از زوایای قاعده مثلث داخلی از زاویه نظیرش (؟) در مثلث بیرونی بزرگتر باشه!
البته اينارو من براي شهود ميگم و حتي حل اين مشكلات به منزله اثبات نيست!به نظر من بايد يكم دقيقتر فكر و صحبت كنيد، يعني يكم به زبون رياضي!
بررسي حالتهاي ممكنه از نظر خودمون ، و بدترين حالتها از نظر خودمون ، و ديدن درستي حكم در مورد اون حالتها ، براي ايده و شهود گرفتن خیلی خوبه ، اما اثبات نيست.

Kesel
13-08-2012, 21:12
خب نه من فک کنم منظورم رو بد رسوندم یا در واقع واضح نرسوندم.
ببینید من متوجهم شما چی می فرمایید ، منظورتون همچین چیزی هست دیگه :

[ برای مشاهده لینک ، لطفا با نام کاربری خود وارد شوید یا ثبت نام کنید ]
الان اگر از F بر BC عمود کنیم ، کوچکتر می شه از این که از A به BC عمود کنیم در حالی که قسمتی از T2 خارج از T1 هست .
من این رو فک می کنم اشاره کردم شاید واضح نگفتم که اگه به همچین شکلی برخوردیم ، این رو می تونیم با دوران دادن ، درون مثلث T1 بندازیم . چرا ؟ چون الان همون چیزی پیش اومده که من اول اثباتم با شرط هایی که گذاشتم رامش کردم :


می دونیم که هم دوران و هم انتقال ایزومتری هستند . یعنی فاصله ی نقاط رو تغییر نمی دن و در نتیجه محیط تغییری نمی کنه . یکی از رئوس غیر منفرجه ی مثلث T2 رو انتقال می دیم روی یکی از اضلاع T1 (مهم نیست کدوم ضلع). حالا حول همون راس ، مثلث T2 رو دوران می دیم تا ضلع رو به رو به زاویه ی منفرجه یا قائمه (اگر زاویه ی منفرجه یا قائمه داشتیم در غیر این صورت ضلع رو به رو به هر زاویه ای) ، منطبق بر ضلع مثلث T1 بشه

ببینید یعنی اون زاویه ی DEF طبق شرطی که گذاشتم حق نداره منفرجه باشه . به محض این که منفرجه شد دورانش می دیم تا راس منفرجه روی BC نیفته

بنابراین این قسمت رو با شرطی که گذاشته بودم کاملا می شه برطرف کرد

حالا اگه منفرجه نبود شکلی غیر از شکل زیر نخواهیم داشت :
[ برای مشاهده لینک ، لطفا با نام کاربری خود وارد شوید یا ثبت نام کنید ]


این رو هم من پیش بینی کرده بودم . این بار نیاز به انجام عمل انتقال داریم .

[ برای مشاهده لینک ، لطفا با نام کاربری خود وارد شوید یا ثبت نام کنید ]

فقط دو حالت وجود داره که مثلث T2 ارتفاعی کوچک تر از ارتفاع T1 داشته باشه ، ولی کل T2 درون T1 نباشه . اولین حالت رو می شه با دوران و دومی رو هم با انتقالمی شه استاندارد کرد.

حالتی که هر سه ارتفاع T2 بزرگتر از T1 است در حالی که T2 درون T1 باشد ، محال است ولی من نیازی به اثبات محال بودنش ندارم چون بر فرض محال ، همونطوری که توی پست قبل اشاره کردم ، خدشه ای به حکم وارد نمی کنه.

بنابراین اگر شما ضلع BC رو محور x ها در نظر بگیرید فقط در صورتی مثلث T2 بیرون قرار خواهد گرفت که نقطه ای روی آن ، عرض بیشتری نسبت به بالاترین نقطه ی T1 داشته باشد. در غیر این صورت می شه با شروطی که گذاشتم مرتفعشون کرد .

حرف من رو از لحاظ شهودی قبول دارید ؟ اگر قبول دارید من حاضرم از لحاظ ریاضی هم روش فکر کنم که با فرمول این بحث رو ثابت کنم
ممنون

ali_hp
14-08-2012, 21:36
خب نه من فک کنم منظورم رو بد رسوندم یا در واقع واضح نرسوندم.
ببینید من متوجهم شما چی می فرمایید ، منظورتون همچین چیزی هست دیگه :

[ برای مشاهده لینک ، لطفا با نام کاربری خود وارد شوید یا ثبت نام کنید ]
الان اگر از F بر BC عمود کنیم ، کوچکتر می شه از این که از A به BC عمود کنیم در حالی که قسمتی از T2 خارج از T1 هست .
من این رو فک می کنم اشاره کردم شاید واضح نگفتم که اگه به همچین شکلی برخوردیم ، این رو می تونیم با دوران دادن ، درون مثلث T1 بندازیم . چرا ؟ چون الان همون چیزی پیش اومده که من اول اثباتم با شرط هایی که گذاشتم رامش کردم :


ببینید یعنی اون زاویه ی DEF طبق شرطی که گذاشتم حق نداره منفرجه باشه . به محض این که منفرجه شد دورانش می دیم تا راس منفرجه روی BC نیفته

بنابراین این قسمت رو با شرطی که گذاشته بودم کاملا می شه برطرف کرد
با توجه به ادامه استدلالتون ، شما مجاز به استفاده از شرطهایی که برای رام کردن گذاشتین ، نیستین!یعنی اینکه شما می خواین استدلالتدنو برای هر سه ارتفاع تعمیم بدین ، و بگین که هر ارتفاع مثلث داخلی "متناظرا" از یک ارتفاع مثلث بیرونی بزرگتره.(و اینجا هم باید بیشتر دقت کنید!حتی اینکه ثابت کنید همه ارتفاعهای مثلث داخلی از "یک" ارتفاع مثلث بیرونی بزرگتره ، کافی نیست!) پس مثلا الان استدلالتون ارتفاع خارج شده از F رو پوشش نمیده.
لطفا کمی با دقت بیشتر ادامه بدین!مشکل همون چیزیه که اول گفتم ، این ایجاب کردن واضح نیست ، و اثباتی هم براش ارایه نکردین!

حالا اگه منفرجه نبود شکلی غیر از شکل زیر نخواهیم داشت :



این رو هم من پیش بینی کرده بودم . این بار نیاز به انجام عمل انتقال داریم .

[ برای مشاهده لینک ، لطفا با نام کاربری خود وارد شوید یا ثبت نام کنید ]

فقط دو حالت وجود داره که مثلث T2 ارتفاعی کوچک تر از ارتفاع T1 داشته باشه ، ولی کل T2 درون T1 نباشه . اولین حالت رو می شه با دوران و دومی رو هم با انتقالمی شه استاندارد کرد.

حالتی که هر سه ارتفاع T2 بزرگتر از T1 است در حالی که T2 درون T1 باشد ، محال است ولی من نیازی به اثبات محال بودنش ندارم چون بر فرض محال ، همونطوری که توی پست قبل اشاره کردم ، خدشه ای به حکم وارد نمی کنه.

بنابراین اگر شما ضلع BC رو محور x ها در نظر بگیرید فقط در صورتی مثلث T2 بیرون قرار خواهد گرفت که نقطه ای روی آن ، عرض بیشتری نسبت به بالاترین نقطه ی T1 داشته باشد. در غیر این صورت می شه با شروطی که گذاشتم مرتفعشون کرد .

حرف من رو از لحاظ شهودی قبول دارید ؟ اگر قبول دارید من حاضرم از لحاظ ریاضی هم روش فکر کنم که با فرمول این بحث رو ثابت کنم
ممنون
مثالتون خیلی خاصه ، لزومی نداره راسها بر هم منطبق باشن.و البته:

بخصوص وقتي قاعده مثلث داخلي خيلي جاي مانور نداشته باشه و یکی از زوایای قاعده مثلث داخلی از زاویه نظیرش (؟) در مثلث بیرونی بزرگتر باشه!
شکلهایی که شما رسم کردین فقط ویژگی دوم از این دو ویژگی خاصو داره ، حالا اگه قاعده مثلث داخلی جای مانور نداشته باشه ، یعنی بزرگ باشه ،(تقریبا هم اندازه ضلع نظیرش در مثلث بیرونی!) دیگه اصلا واضح نیست که میشه با انتقال مشکل حل کرد.(البته شما هم اثباتی یا شهودی! از اینکه میشه با انتقال مشکلو حل کرد ارایه نکردین.)
باز هم میگم ، به نظر من باید دقیقتر فکر کنید ، و سعی کنید چیزهایی که به نظرتون درسته رو اثبات کنید...
نه ، من از نظر شهودی هم قبول ندارم حرفتونو .
این مساله که ایا میشه واقعا یک مثلث داخل مثلث دیگرو با انتقال و دوران یک ضلعشو انداخت روی مثلث بیرونی مساله جالبیه.البته مساله ای که شما برای تکمیل راه حلتون نیاز دارید دقیقا این نیست!فقط اگه بتونین مثلثی همنهشت با مثلث داخلی رو جایی که مد نظرتون هست رسم کنید ، حله!
و البته راه حلی هم براش ندارم.به نظرم خیلی ساده نباشه!
اگر فقط هدفتون این باشه که دو راس مثلث داخلی بیافتن روی اضلاع مثلث بیرونی (نه لزوما روی یک ضلع ) کاره سختی نیست ، ولی ادامه راه حل هم عوض میشه یکم!

Kesel
14-08-2012, 21:58
من فکر می کنم منظورم رو کامل متوجه نشدید . ولی خب باز هم مهم نیست چون شما به چیزی اشاره کردید که دیگه به نظرم لازم نیست استدلالم رو توضیح بدم :



شکلهایی که شما رسم کردین فقط ویژگی دوم از این دو ویژگی خاصو داره ، حالا اگه قاعده مثلث داخلی جای مانور نداشته باشه ، یعنی بزرگ باشه ،(تقریبا هم اندازه ضلع نظیرش در مثلث بیرونی!)


بله این مساله رو تصور کرده بودم منتهی تصویر سازی ذهنیم اشتباه بود ، در این حالتی که گفتید بی نهایت مثلث قابل رسمن که صحبت من رو نقض می کنن ، یعنی مشکلشون با انتقال و دوران حل نمی شه .
بنابراین استدلال من کامل نیست و غیر قابل ارائه هست مگر این که کامل بشه (که حتما متوجه شدید چه قدر دشوار هست سروکله زدن با مثلثی که حتی با این همه شرط باز هم رام نمی شه)

در هر حال هر دو راه حل شما به نظر من درست هستند و راحت و قابل فهم

ممنون

nft
10-10-2012, 23:21
سلام هركي ميتونه لطف كنه اثبات كنه :
بين مثلث هايي با محيط برابر مثلث متساوي الاضلاع بيشترين مساحت را دارد
تورو خدا زودتر جواب بدين لازم دارم
مرسي از سايت باحال بروبچ باحالترش

fatholahi
11-10-2012, 11:04
سلام دوستان من سوالی واسم پیش اومده رشته من ریاضی نیست اگر دوستان می تونن کمک کنن ممنون می شم.
سوال

مختصات راس مخروط = (x1, y1, h1)
زاویه مخروط = alpha
ارتفاع مخروط = H
شعاع مخروط = R
مختصات یک نقطه درون مخروط = P1 (x2, y2, h2)
مختصات یک نقطه بیرون از مخروط = P2( x3, y3, h3)

من فرمولی می خوام که از روی اون تشخیص بدم که ایا یک نقطه با مختصات داده شده در داخل مخروط است یا خیر
مثلا فرمول برای نقطه P1 باید تشخیص بده داخله مخروطه اما نقطه دیگه بگه داخل مخروط نیست
ممنونم از توجه شما

Kesel
11-10-2012, 12:25
سلام هركي ميتونه لطف كنه اثبات كنه :
بين مثلث هايي با محيط برابر مثلث متساوي الاضلاع بيشترين مساحت را دارد
تورو خدا زودتر جواب بدين لازم دارم
مرسي از سايت باحال بروبچ باحالترش

سلام
اثبات ساده ای بر پایه ی قضایای « هرون » و « نابرابری میانگین حسابی-هندسی » داره :

اثبات :

بنابر قضیه ی هرون داریم : (توجه کنید که S مساحت مثلث و P نصف محیط مثلث است)

[ برای مشاهده لینک ، لطفا با نام کاربری خود وارد شوید یا ثبت نام کنید ]^{2}=p(p-a)(p-b)(p-c)

چون P عددی ثابت است لذا برای حداکثر کردن مساحت باید سمت راست تساوی زیر حداکثر شود:

[ برای مشاهده لینک ، لطفا با نام کاربری خود وارد شوید یا ثبت نام کنید ]{S^{2}}{p}=(p-a)(p-b)(p-c)

بنا به قضیه ی نابرابری میانگین حسابی-هندسی داریم :

[ برای مشاهده لینک ، لطفا با نام کاربری خود وارد شوید یا ثبت نام کنید ]{S^{2}}{p}=(p-a)(p-b)(p-c)\leq&space;\left&space;(&space;\frac{(p-a)&plus;(p-b)&plus;(p-c)}{3}&space;\right&space;)^{3}

و باز هم بنا به قضیه ی نابرابری میانگین حسابی-هندسی می دانیم که تساوی فقط در صورتی اتفاق خواهد افتاد که :

[ برای مشاهده لینک ، لطفا با نام کاربری خود وارد شوید یا ثبت نام کنید ](p-a)=(p-b)=(p-c)\Leftrightarrow&space;a=b=c

لذا از تمام مثلث هایی با محیط برابر ، این مثلث متساوی الاضلاع است که بیشترین مساحت را دارد.

فرمول هرون :

برای مشاهده محتوا ، لطفا وارد شوید یا ثبت نام کنید

نابرابری میانگین حسابی-هندسی :

برای مشاهده محتوا ، لطفا وارد شوید یا ثبت نام کنید

Kesel
11-10-2012, 13:30
سلام دوستان من سوالی واسم پیش اومده رشته من ریاضی نیست اگر دوستان می تونن کمک کنن ممنون می شم.
سوال

مختصات راس مخروط = (x1, y1, h1)
زاویه مخروط = alpha
ارتفاع مخروط = H
شعاع مخروط = R
مختصات یک نقطه درون مخروط = P1 (x2, y2, h2)
مختصات یک نقطه بیرون از مخروط = P2( x3, y3, h3)

من فرمولی می خوام که از روی اون تشخیص بدم که ایا یک نقطه با مختصات داده شده در داخل مخروط است یا خیر
مثلا فرمول برای نقطه P1 باید تشخیص بده داخله مخروطه اما نقطه دیگه بگه داخل مخروط نیست
ممنونم از توجه شما

من منظورتون رو از داخل مخروط متوجه نمی شم ... داخل یک رویه ی درجه ی دوم چه معنایی داره ؟

در هر حال برای این که ببینیم نقطه ای با طول و عرض و ارتفاع x , y , z جزو مخروط هست یا نه باید این نقطه رو در معادله ی مخروط بزارید . اگر صدق کرد این نقطه جزئی از مخروطه در غیر این صورت نیست

NFT000
12-10-2012, 17:50
عالي بود واقعا ممنون

من واقعا از KESEL ممنونم چون واقغا احتياج داشتم به جوابش



سلام
اثبات ساده ای بر پایه ی قضایای « هرون » و « نابرابری میانگین حسابی-هندسی » داره :

اثبات :

بنابر قضیه ی هرون داریم : (توجه کنید که S مساحت مثلث و P نصف محیط مثلث است)

[ برای مشاهده لینک ، لطفا با نام کاربری خود وارد شوید یا ثبت نام کنید ]^{2}=p(p-a)(p-b)(p-c)

چون P عددی ثابت است لذا برای حداکثر کردن مساحت باید سمت راست تساوی زیر حداکثر شود:

[ برای مشاهده لینک ، لطفا با نام کاربری خود وارد شوید یا ثبت نام کنید ]{S^{2}}{p}=(p-a)(p-b)(p-c)

بنا به قضیه ی نابرابری میانگین حسابی-هندسی داریم :

[ برای مشاهده لینک ، لطفا با نام کاربری خود وارد شوید یا ثبت نام کنید ]{S^{2}}{p}=(p-a)(p-b)(p-c)\leq&space;\left&space;(&space;\frac{(p-a)&plus;(p-b)&plus;(p-c)}{3}&space;\right&space;)^{3}

و باز هم بنا به قضیه ی نابرابری میانگین حسابی-هندسی می دانیم که تساوی فقط در صورتی اتفاق خواهد افتاد که :

[ برای مشاهده لینک ، لطفا با نام کاربری خود وارد شوید یا ثبت نام کنید ](p-a)=(p-b)=(p-c)\Leftrightarrow&space;a=b=c

لذا از تمام مثلث هایی با محیط برابر ، این مثلث متساوی الاضلاع است که بیشترین مساحت را دارد.

فرمول هرون :

برای مشاهده محتوا ، لطفا وارد شوید یا ثبت نام کنید

نابرابری میانگین حسابی-هندسی :

برای مشاهده محتوا ، لطفا وارد شوید یا ثبت نام کنید
سلام
ميخواستم بپرسم از راه بهينه سازي هم حل ميشه ؟؟؟؟؟؟؟؟؟؟؟
اگه بشه واقعا عاليه
از همه علاقمندان ممنوووونم

daniel300
14-10-2012, 10:29
سلام دوستان

2 تا سوال فوری داشتم از هندسه هیلبرت !

1-CAB> و نقطه D واقع بر BC مفروض اند.انگاه D در درون CAB> واقع است،اگر و فقط اگر B*D*C (* به معنای میان است،از اصطلاحات تعریف نشده.)


2-قضیه قطعه بر : هرگاه AD بین AC و AB واقع باشد انگا AD پاره خط BC را می برد.

ایا کسی میتونه با استفاده از بنداشتهای هیلبرت 1 گزاره و یک قضیه را ثابت کند؟ !

ممنون دوستان

Kesel
14-10-2012, 14:35
سلام دوستان

2 تا سوال فوری داشتم از هندسه هیلبرت !

1-CAB> و نقطه D واقع بر BC مفروض اند.انگاه D در درون CAB> واقع است،اگر و فقط اگر B*D*C (* به معنای میان است،از اصطلاحات تعریف نشده.)


2-قضیه قطعه بر : هرگاه AD بین AC و AB واقع باشد انگا AD پاره خط BC را می برد.

ایا کسی میتونه با استفاده از بنداشتهای هیلبرت 1 گزاره و یک قضیه را ثابت کند؟ !

ممنون دوستان

سلام
جواب هر دو سوالتون و سوالات مشابه رو در پی دی اف زیر جستجو کنید :


برای مشاهده محتوا ، لطفا وارد شوید یا ثبت نام کنید

بیان قضیه و اثبات سوال اولتون تحت عنوان Lemma 3.6 و بیان و اثبات سوال دومتون تحت عنوان Theorem 3.10 در این پی دی اف مطرح شده

موفق باشید

daniel300
14-10-2012, 16:35
سلام
جواب هر دو سوالتون و سوالات مشابه رو در پی دی اف زیر جستجو کنید :


برای مشاهده محتوا ، لطفا وارد شوید یا ثبت نام کنید

بیان قضیه و اثبات سوال اولتون تحت عنوان Lemma 3.6 و بیان و اثبات سوال دومتون تحت عنوان Theorem 3.10 در این پی دی اف مطرح شده

موفق باشید



دوست عزیز یک دنیا ممنون.

mr.badiei76
15-10-2012, 22:37
يه سوال
ثابت كنيد اگر در مثلثي دو ميانه با هم برابر باشند آن مثلث متساوي الساقين است.

Kesel
16-10-2012, 00:08
يه سوال
ثابت كنيد اگر در مثلثي دو ميانه با هم برابر باشند آن مثلث متساوي الساقين است.

البته که این قضیه دو شرطی هست.

اثبات :

مثلث زیر را در نظر بگیرید :

[ برای مشاهده لینک ، لطفا با نام کاربری خود وارد شوید یا ثبت نام کنید ]

بنا به فرض CD=BE . پاره خط FB را هم اندازه ی ED در امتداد BC رسم می کنیم .
نقطه ی D را به F وصل می کنیم . لذا DEBF متوازی الاضلاع است . ( بنا بر یک قضیه ، خط واصل نقاط میانی دو ضلع ، با ضلع سوم موازی است )
طبق قضیه ی خطوط موازی و مورب ، زاویه ی F و B1 که با a نشان داده شده اند با هم برابرند.
اضلاع رو به رو در متوازی الاضلاع با هم برابرند لذا : FD=BE که طبق فرض : BE=CD بنابراین خواهیم داشت : FD=DC که نشان دهنده ی متساوی الساقین بودن مثلث FDC است پس : C1=a

BC مشترک
B1=C1
BE=DC

لذا دو مثلث BEC و CDB به حالت دو ضلع و زاویه ی بین همنهشتند . نتیجه ی زیر از همنهشتی دو مثلث به دست می آید :
B=C (زوایای B و C مساوی هستند)
که عبارت بالا حکم را مستقیما اثبات می کند

hamid1002
09-11-2012, 07:42
با عرض سلام
ابتدا اینکه اگه سوالمو جای نامناسبی مطرح میکنم از همگی عذار میخوام هرچقدر دنبال گشتم تاپیکی که مربوط به سوالم بشه رو پیدا نکردم

اما سوالم از دوستان عزیز اینه:
لطفا اثبات فرمول مدول برشی قانون هوک (مدول برشیGو vضریب پواسون) رو بصورت کامل بذارید

G=E/2 (1+v

با تشکر

Kesel
09-11-2012, 10:49
با عرض سلام
ابتدا اینکه اگه سوالمو جای نامناسبی مطرح میکنم از همگی عذار میخوام هرچقدر دنبال گشتم تاپیکی که مربوط به سوالم بشه رو پیدا نکردم

اما سوالم از دوستان عزیز اینه:
لطفا اثبات فرمول مدول برشی قانون هوک (مدول برشیGو vضریب پواسون) رو بصورت کامل بذارید

G=E/2 (1+v

با تشکر


سلام
متاسفانه من اطلاعاتی در این زمینه ندارم اما لینک زیر رو پیدا کردم ببینید به دردتون می خوره :


برای مشاهده محتوا ، لطفا وارد شوید یا ثبت نام کنید

hamid1002
09-11-2012, 23:47
ممنون دوست عزیز ولی من بصورت کاملشو میخوام این اخرشو اینقدر تند رفته که نمیشه متوجه شد

handroid
19-11-2012, 20:02
دوستان ممنون میشم همرسی میانه ها و ارتفا هارو واسم اثبات کنید من که الان 1ساعته دارم فک میکنم:|لطفا امروز ج بدید فردا امتحان دارام

esfhost
08-12-2012, 22:59
چرا وقتی دو متغیر تصادفی مستقل اند هرترکیبی از اون ها با یکیشون مستقل اند چرا وقتی دو متغیر تصادفی مستقل اند هرترکیبی از اون ها با یکیشون مستقل اند؟؟؟
Sent from my GT-I9100 using Tapatalk 2

armanassassin
08-12-2012, 23:48
اگه کسی میتونه به اینا جواب بده ممنون میشم
الف( اگر صفحه را با سه رنگ مختلف رنگ کنیم، ثابت کنید دو نقطهی همرنگ با فاصلهی 1 وجود دارند.
ب( اگر فضا را با چهار رنگ مختلف رنگ کنیم، ثابت کنید دو نقطهی همرنگ با فاصلهی 1 وجود دارند.
اگه تونستین سریع لطفا
بی زحمت کسایی که هستن اگه میتونن جواب مارو بدن

fht_009
09-12-2012, 17:04
سلام
یه سوال ابتدایی دارم از هندسه ،ممنون میشم جواب بدید.

ثابت کنید در هر مثلث متساوی الساقین :
الف)دو زاویه مربوط به ساق ها با هم برابرند
ب)ارتفاع،میانه،عمودمنصف و نیمساز در مثلث نتساوی الساقین به هم منطبق اند



ممنون

flashdesign
14-04-2013, 13:57
سلام و درود بر شما
من یه سوال دارم ک برای درس مبانی هندسه دانشگاهه
البته ااین موضوع ب عنوان راهنمایی برای حل یه سوال دیگه مطرح شده ولی من قادر ب حلش نیستم
در واقع نمیدونم چطوری میشه یک به یک بودن این رو حساب کرد در صورتی ک دوتا متغیر داره یعنی باید یک ب یک بودن تابع زیر رو بررسی کنیم.
ممنون میشم اگر راهنمایی کنید.
[ برای مشاهده لینک ، لطفا با نام کاربری خود وارد شوید یا ثبت نام کنید ]

Kesel
18-04-2013, 10:40
سلام و درود بر شما
من یه سوال دارم ک برای درس مبانی هندسه دانشگاهه
البته ااین موضوع ب عنوان راهنمایی برای حل یه سوال دیگه مطرح شده ولی من قادر ب حلش نیستم
در واقع نمیدونم چطوری میشه یک به یک بودن این رو حساب کرد در صورتی ک دوتا متغیر داره یعنی باید یک ب یک بودن تابع زیر رو بررسی کنیم.
ممنون میشم اگر راهنمایی کنید.
[ برای مشاهده لینک ، لطفا با نام کاربری خود وارد شوید یا ثبت نام کنید ]


سلام

من یک به یک بودن رو برای توابع دو متغیره نشنیدم . یه نگاهی هم توی اینترنت انداختم چیزی پیدا نکردم .

به شکل زیر توجه کنید :

[ برای مشاهده لینک ، لطفا با نام کاربری خود وارد شوید یا ثبت نام کنید ](\frac{x}{y})

[ برای مشاهده لینک ، لطفا با نام کاربری خود وارد شوید یا ثبت نام کنید ]

کد متمتیکا :


برای مشاهده محتوا ، لطفا وارد شوید یا ثبت نام کنید

اینو می شه واقعا گفت یک به یکه ؟ دلیل این که یه معیار رسمی برای قضاوت در مورد یک به یک بودن توابع دو متغیره نداریم اینه که معرفی « یک به یک بودن » برای توابع با بیش از یک متغیر تعمیم تعریف یک به یک بودنه . بنابراین شاید بشه گفت تابعی یک به یکه که به ازای هر مقدارش فقط و فقط یک نقطه متناظر باهاش از صفحات موازی با x-y وجود داشته باشه . یا به عبارت ساده تر هر صفحه ی موازی با صفحه ی x-y ، نمودار تابع رو فقط در یک نقطه قطع کنه که نمی کنه (مطابق شکل)

جایی هم دیدم نوشته بود :

[ برای مشاهده لینک ، لطفا با نام کاربری خود وارد شوید یا ثبت نام کنید ]{(x,y)}=f_{(x',y')}\;&space;\ ;&space;\;&space;\;&space;Then\;&space;\;&space;\;&space;\;&space;x=x',y=y'

در هر حال وقتی کسی تعریفی رو تعمیم می ده شاید بهتر باشه اونو بازنویسی کنه .

شما خواستین سوالو بزارین شاید بهتر بشه کمک کرد.

flashdesign
18-04-2013, 14:17
سلام

شما خواستین سوالو بزارین شاید بهتر بشه کمک کرد.

سوال1:
گزاره "خطوط موازی یکدیگر را در بینهایت قطع میکنند" را در هر سه مدل معرفی شده در این بخش توضیح دهید .در صفحه پوانکاره و یا کره ریمان کلمه "در بینهایت " معنی دارد؟

سوال 2
ثابت کنید:
[ برای مشاهده لینک ، لطفا با نام کاربری خود وارد شوید یا ثبت نام کنید ]در حالت[ برای مشاهده لینک ، لطفا با نام کاربری خود وارد شوید یا ثبت نام کنید ] در شرط دوم تعریف d صدق میکند.

در واقع جواب هر دو سوال به راهنمایی ک قبلا خواستم مربوط میشه:n13:
تا جایی ک من میدونم برای اثبات یک ب یک بودن f(x,y)f باید c , r را ثابت فرض کنیم و بعد یک ب یک بودنش رو اثبات کنیم.

Kesel
18-04-2013, 16:17
همونطورکه حدس می زدم سوالات شما ورای معلومات منه . اما شما هم فک کنم سوالاتونو کامل نپرسیدین.



گزاره "خطوط موازی یکدیگر را در بینهایت قطع میکنند" را در هر سه مدل معرفی شده در این بخش توضیح دهید

کدوم سه مدل مشخص شده ؟

دومی هم اصن متوجه نمی شم چیه . اگر کامل نوشتید بنابراین معلومات من کفایت فهمیدنشو نمی کنه .

ضمنا توی پست قبلی من هم r و c رو ثابت فرض کردم اگه نه که متغیر ها 4 تا می شد.شما اگه تعریف یک به یک بودن توابع چند متغیره رو پیدا کردید خیلی راحت می تونین در موردش قضاوت کنین.منم باز دنبالش می گردم.

flashdesign
18-04-2013, 16:57
همونطورکه حدس می زدم سوالات شما ورای معلومات منه . اما شما هم فک کنم سوالاتونو کامل نپرسیدین.



کدوم سه مدل مشخص شده ؟

دومی هم اصن متوجه نمی شم چیه . اگر کامل نوشتید بنابراین معلومات من کفایت فهمیدنشو نمی کنه .

ضمنا توی پست قبلی من هم r و c رو ثابت فرض کردم اگه نه که متغیر ها 4 تا می شد.شما اگه تعریف یک به یک بودن توابع چند متغیره رو پیدا کردید خیلی راحت می تونین در موردش قضاوت کنین.منم باز دنبالش می گردم.


ممنون از دقت و توجه شما
سوالات رو عیناً از روی سوالاتی ک بهمون دادن نوشتم (بدون تغییر ) اتفاقا یکی از دلایلی ک نمیتونم حلش کنم اینه ک سوال رو درست متوجه نشدم.
چند تابع دیگه هم هستن ک باید یک ب یک و پوشا بودنشون رو مشخص کنم و اونها هم دو متغیره هستن و دارم روشون فکر میکنم ولی هنوز ب نتیجه خاصی نرسیدم.
باز هم سپاس فراوان برای وقتی ک گذاشتید و همینطور برای راهنمایی شما:n16:

ali_hp
20-04-2013, 02:39
سلام

من یک به یک بودن رو برای توابع دو متغیره نشنیدم . یه نگاهی هم توی اینترنت انداختم چیزی پیدا نکردم .

به شکل زیر توجه کنید :

[ برای مشاهده لینک ، لطفا با نام کاربری خود وارد شوید یا ثبت نام کنید ](\frac{x}{y})

[ برای مشاهده لینک ، لطفا با نام کاربری خود وارد شوید یا ثبت نام کنید ]

کد متمتیکا :


برای مشاهده محتوا ، لطفا وارد شوید یا ثبت نام کنید

اینو می شه واقعا گفت یک به یکه ؟ دلیل این که یه معیار رسمی برای قضاوت در مورد یک به یک بودن توابع دو متغیره نداریم اینه که معرفی « یک به یک بودن » برای توابع با بیش از یک متغیر تعمیم تعریف یک به یک بودنه . بنابراین شاید بشه گفت تابعی یک به یکه که به ازای هر مقدارش فقط و فقط یک نقطه متناظر باهاش از صفحات موازی با x-y وجود داشته باشه . یا به عبارت ساده تر هر صفحه ی موازی با صفحه ی x-y ، نمودار تابع رو فقط در یک نقطه قطع کنه که نمی کنه (مطابق شکل)

جایی هم دیدم نوشته بود :

[ برای مشاهده لینک ، لطفا با نام کاربری خود وارد شوید یا ثبت نام کنید ]{(x,y)}=f_{(x',y')}\;&space;\ ;&space;\;&space;\;&space;Then\;&space;\;&space;\;&space;\;&space;x=x',y=y'

در هر حال وقتی کسی تعریفی رو تعمیم می ده شاید بهتر باشه اونو بازنویسی کنه .

شما خواستین سوالو بزارین شاید بهتر بشه کمک کرد.
سلام دوستان
واقعا یک معیار رسمی داریم!اصلا تعریف تابع و یک به یک بودن روی مجموعه ها صورت می گیره، حالا اگه اعضای مجموعه دامنه ما زوج مرتب باشن، همون چیزی میشه که بهش می گیم تابع دو متغیره!البته تعمیم شما از یک به یک بودن هم کاملا درسته، واین تابع به همون دلیلی که شما گفتی یک به یک نیست. ولی خوب اگه تابع رو روی زیر مجموعه های کوچکتری از R^2 در نظر بگیریم ممکنه یک به یک بشه، که احتمالا در صورت سوال جا افتاده!
این تابع که ظاهرا از R^2 تعریف شده به R ، یعنی ورودی تابع به صورت زوج مرتب (x,y) هست، برای اثبات یک به یک بودن تابع نیز باید ثابت کنیم:
اگر f(x,y)=f(a,b)l انگاه (x,y)=(a,b) یا به عبارت دیگر x=a و y=b .

flashdesign
20-04-2013, 17:56
سلام دوستان
واقعا یک معیار رسمی داریم!اصلا تعریف تابع و یک به یک بودن روی مجموعه ها صورت می گیره، حالا اگه اعضای مجموعه دامنه ما زوج مرتب باشن، همون چیزی میشه که بهش می گیم تابع دو متغیره!البته تعمیم شما از یک به یک بودن هم کاملا درسته، واین تابع به همون دلیلی که شما گفتی یک به یک نیست. ولی خوب اگه تابع رو روی زیر مجموعه های کوچکتری از R^2 در نظر بگیریم ممکنه یک به یک بشه، که احتمالا در صورت سوال جا افتاده!
این تابع که ظاهرا از R^2 تعریف شده به R ، یعنی ورودی تابع به صورت زوج مرتب (x,y) هست، برای اثبات یک به یک بودن تابع نیز باید ثابت کنیم:
اگر f(x,y)=f(a,b)l انگاه (x,y)=(a,b) یا به عبارت دیگر x=a و y=b .

سلام و درود بر شما
ممنونم واقعا
با توجه به راهنمایی شما دقیقا تونستم مثالهایی ک درباره این موضوع بود ب راحتی حل کنم.
باز هم سپاس:n16:

skyzare
27-04-2013, 19:50
با سلام .

اساتید این فرمول تفاضل و جمع دو بردار که به صورت زیر هست چه جوری قابل اثبات هست ؟



[ برای مشاهده لینک ، لطفا با نام کاربری خود وارد شوید یا ثبت نام کنید ][2]{A^2&plus;B^2&plus;2ABcos\:&space;\theta&space;}\\\\\\&space;A-B=R=\sqrt[2]{A^2&plus;B^2-2ABcos\:&space;\theta&space;}

*M!L4D*
28-04-2013, 03:48
با سلام .

اساتید این فرمول تفاضل و جمع دو بردار که به صورت زیر هست چه جوری قابل اثبات هست ؟



[ برای مشاهده لینک ، لطفا با نام کاربری خود وارد شوید یا ثبت نام کنید ][2]{A^2&plus;B^2&plus;2ABcos\:&space;\theta&space;}\\\\\\&space;A-B=R=\sqrt[2]{A^2&plus;B^2-2ABcos\:&space;\theta&space;}


سلام
شما اگه دو بردار رسم کنید می بینید که یک متوازی اضلاع تشکیل میشه ( اگه هم راستا نباشند ) . برآیند و تفاضل این دو بردار همون اقطار متوازی الاضلاع هستند که با استفاده از قضیه کسینوس ها بدست میان .
اثبات قضیه کسینوس ها :
[ برای مشاهده لینک ، لطفا با نام کاربری خود وارد شوید یا ثبت نام کنید ]

Hamid2545
14-05-2013, 22:28
ثابت کنید در هر مثلث : محل تقاطع میانه ها , محل تقاطع عمود منصف ها , محل تقاطع ارتفاع ها و محل تقاطع نیمسازها همگی بر یک امتدادند

elena2022
06-06-2013, 15:42
میشه لطف کنین اینو واسه من اثبات کنین
|w|+|z|≥|z+w|

flashdesign
06-06-2013, 19:36
میشه لطف کنین اینو واسه من اثبات کنین
|w|+|z|≥|z+w|
سوالتون کامل نیست
نگفتید که zوwچی هستن؟
با اینحال امیدوارم اینایی که میگم به دردتون بخوره

تعبیر هندسی

[ برای مشاهده لینک ، لطفا با نام کاربری خود وارد شوید یا ثبت نام کنید ]

حالا اثبات
[ برای مشاهده لینک ، لطفا با نام کاربری خود وارد شوید یا ثبت نام کنید ]

elena2022
07-06-2013, 00:29
مرسی از لطفتون
بله یادم رفت بگم این سوال مربوط به اعداد مختلطه
[ برای مشاهده لینک ، لطفا با نام کاربری خود وارد شوید یا ثبت نام کنید ] ([ برای مشاهده لینک ، لطفا با نام کاربری خود وارد شوید یا ثبت نام کنید ])

flashdesign
07-06-2013, 07:51
مرسی از لطفتون
بله یادم رفت بگم این سوال مربوط به اعداد مختلطه
[ برای مشاهده لینک ، لطفا با نام کاربری خود وارد شوید یا ثبت نام کنید ] ([ برای مشاهده لینک ، لطفا با نام کاربری خود وارد شوید یا ثبت نام کنید ])

کلا خسته نباشی شما:n02:

elena2022
07-06-2013, 14:42
چرا آخه :n13:

flashdesign
07-06-2013, 15:45
چرا آخه :n13:
آخه چون نگفته بودید من فک کردم واسه اعداد طبیعی و ایناس اصلا تو فکره اعداد مختلط نبودم و فک کنم اصلا جوابم به کارتون نیاد...
اگه دوستان جواب ندادن انشالله فردا اگه بتونم جوابش رو میابم براتون
راستی سوال مال هندسه هستش؟
سوال کامل رو گفتید همونه؟
چیز دیگه ای درباره سوال ندادن؟
در سطح دانشگاهه؟
مال کتابه یا استاد تمرین داده؟

elena2022
07-06-2013, 17:01
بله این دفعه دیگه کامل گفتم[ برای مشاهده لینک ، لطفا با نام کاربری خود وارد شوید یا ثبت نام کنید ]
بله مال دانشگاه هست
نه تمرین استاده
ممنون که جواب دادین[ برای مشاهده لینک ، لطفا با نام کاربری خود وارد شوید یا ثبت نام کنید ]

Kesel
07-06-2013, 18:28
آخه چون نگفته بودید من فک کردم واسه اعداد طبیعی و ایناس اصلا تو فکره اعداد مختلط نبودم و فک کنم اصلا جوابم به کارتون نیاد...
اگه دوستان جواب ندادن انشالله فردا اگه بتونم جوابش رو میابم براتون
راستی سوال مال هندسه هستش؟
سوال کامل رو گفتید همونه؟
چیز دیگه ای درباره سوال ندادن؟
در سطح دانشگاهه؟
مال کتابه یا استاد تمرین داده؟

خب همون پستی که دادین جوابشون بود دیگه چی رو مگه می خواستین اثبات کنین ؟

flashdesign
07-06-2013, 20:39
خب همون پستی که دادین جوابشون بود دیگه چی رو مگه می خواستین اثبات کنین ؟

آخه شرط و شروطی که دادن رو در نظر نگرفته بودم
شما سوال که پرسیده شده بود و جوابی که من دادم رو چک کردید؟
درست بود ؟؟؟؟
امیدوارم درست بوده باشه
ممنون وقت گذاشتید:n16:

Kesel
07-06-2013, 22:01
شرط خاصی نداره مساله شون . فقط گفته z و w مختلط باشن که خب توی اثباتتون این شرط رو در نظر گرفته.
اثبات شما از نامساوی کوشی-شوارتز استفاده کرده . اثبات دیگه (و معمول تری) داره که شاید مفید بود برا سوال کننده ی محترم :

[ برای مشاهده لینک ، لطفا با نام کاربری خود وارد شوید یا ثبت نام کنید ]|z&plus;w|^{2}=(z&plus;w)(\overline{z&plus;w})=(z&plus;w)(\o verline{z}&plus;\overline{w})=|z|^{2}&plus;|w|^{2}&plus;\overline {z}w&plus;z\overline{w}=|z|^{2}&plus;|w|^{2}&plus;2Re(\overline{z }w)\leq&space;|z|^{2}&plus;|w|^{2}&plus;2|Re(\overline{z}w)|\leq&space;| z|^{2}&plus;|w|^{2}&plus;2|\overline{z}w|=|z|^{2}&plus;|w|^{2}&plus;2| \overline{z}||w|&space;=|z|^{2}&plus;|w|^{2}&plus;2|z||w|=(|z|&plus;|w| )^{2}

لذا نتیجه می گیریم :

[ برای مشاهده لینک ، لطفا با نام کاربری خود وارد شوید یا ثبت نام کنید ]|z&plus;w|\leq&space;|z|&plus;|w|

javad2015
03-01-2014, 02:00
یکی زحمت بکشه اثبات قضیه تالس رو بزاره.اثباتش یادم رفته.خیلی نا جور رو مخمه.

javad2015
11-02-2014, 01:39
جواب ما رو که کسی نداد میزارم برای بقیه:
[ برای مشاهده لینک ، لطفا با نام کاربری خود وارد شوید یا ثبت نام کنید ]
[ برای مشاهده لینک ، لطفا با نام کاربری خود وارد شوید یا ثبت نام کنید ]

armanassassin
04-03-2014, 19:51
سلام ببخشید یه سوال داشتم . چطور میتونم بدون استفاده از قضیه ی فیثاغورث و کسینوس ها ثابت کنیم یک مثلث به ضلع های 3 و 4 و 5 قایم الزاویه است ؟

solid_engineer
05-03-2014, 14:57
با سلام


سلام ببخشید یه سوال داشتم . چطور میتونم بدون استفاده از قضیه ی فیثاغورث و کسینوس ها ثابت کنیم یک مثلث به ضلع های 3 و 4 و 5 قایم الزاویه است ؟

اضلاع مثلث را بصورت بردارهایی در یک دستگاه مختصات به شرح زیر در نظر میگیریم:

[ برای مشاهده لینک ، لطفا با نام کاربری خود وارد شوید یا ثبت نام کنید ]

از حل همزمان معادلات 1، 2 و 3 مقادیر b، a و c را بدست آورده و به کمک آن بردارهای B،A و C را مطابق زیر تعیین می کنیم:

[ برای مشاهده لینک ، لطفا با نام کاربری خود وارد شوید یا ثبت نام کنید ]

نهایتاً با استفاده از مفهوم ضرب داخلی میتوان زوایای بین بردارهای B،A و C (که همان اضلاع مثلث هستند) را مطابق زیر تعیین نمود:

[ برای مشاهده لینک ، لطفا با نام کاربری خود وارد شوید یا ثبت نام کنید ]

موفق باشید/

armanassassin
05-03-2014, 23:54
با سلام



اضلاع مثلث را بصورت بردارهایی در یک دستگاه مختصات به شرح زیر در نظر میگیریم:

[ برای مشاهده لینک ، لطفا با نام کاربری خود وارد شوید یا ثبت نام کنید ]

از حل همزمان معادلات 1، 2 و 3 مقادیر b، a و c را بدست آورده و به کمک آن بردارهای B،A و C را مطابق زیر تعیین می کنیم:

[ برای مشاهده لینک ، لطفا با نام کاربری خود وارد شوید یا ثبت نام کنید ]

نهایتاً با استفاده از مفهوم ضرب داخلی میتوان زوایای بین بردارهای B،A و C (که همان اضلاع مثلث هستند) را مطابق زیر تعیین نمود:

[ برای مشاهده لینک ، لطفا با نام کاربری خود وارد شوید یا ثبت نام کنید ]

موفق باشید/
ضمن تشکر از شما اینو باید بگم که این سوال رو دبیره ریاضیمون داده. من این رو از دو روش حل کردم ( که یکیشم شبیه روش شما بود )ولی ایشون قبول نکردن گفتن جواب درسته ولی جواب مورد نظر من نیست چون اولش از فیثاغورث استفاده کردی . گفتن باید از تشابه حلش کنین . ولی ممنون از وقتی که گذاشتین .:n16:

javad2015
06-03-2014, 04:13
منظورش این بوده که بیایم قضیه فیثا غورث رو از راه تشابه اثبات کنیم؟بفرما
[ برای مشاهده لینک ، لطفا با نام کاربری خود وارد شوید یا ثبت نام کنید ]

armanassassin
06-03-2014, 07:33
نه منظورم اینه که قایم الزاویه بودنشو از راه تشابه اثبات کنیم . ممنون از زحماتتون

Sara.Ah
31-03-2014, 12:46
می شه لطفا این سوال رو حل کنید؟
"از نقطه o محل همرسی نیمساز های داخلی مثلث ABC خطی به موازات BC رسم می کنیم تا AB و AC را به ترتیب در M و N قطع کند. اگر MB=5 , NC=7 باشد، طول پاره خط MN چند است؟":n27:

Sara.Ah
31-03-2014, 12:50
نه منظورم اینه که قایم الزاویه بودنشو از راه تشابه اثبات کنیم . ممنون از زحماتتون

جواب آقای javad2015 درسته چون از این راه هم تشابه داریم هم اثبات قائم الزاویه بودن

shayan1222
13-10-2014, 22:32
اینو برام اثبات کنید لطفا


ثابت کنید در مثلث ثایم الزاویه مجموع مربعات میانه ها مساوی سه دوم مربع وتر است

1451233345
11-11-2014, 15:17
سلام خیلی لازمه برام ثابت کنید مجموع فاصله های هر نقطه داخل مثلث از سه راس کوچکتر از مجموع سه ضلع مثلث است

mb1372
20-12-2014, 10:40
سلام دوستان
آیا برای اصل کاوالیر! اثباتی ارائه شده؟
اثباتش غیر ممکنه یا پیدا نشده؟
چون توی یکی از درسام به موردی رسیدم که ازش میشه کاوالیر حجم رو اثبات کرد...احتمالا.

محمد معصومی
27-12-2014, 19:05
با تشکر از شما

marry77
28-12-2014, 16:05
سلااام دوستان
من یه سوال داشتم
تعداد مثلث های موجود در هر مثلث احاطه شده ؟
کلا تعدا مثلث هاییکه میتونیم تو یه مثلث دیگه بکشیم چند تاست؟
بی نهایته ؟یه کسی میگه جوابش میشه 4320؟؟؟؟؟؟؟؟؟:n36:

mb1372
31-12-2014, 23:48
سلااام دوستان
من یه سوال داشتم
تعداد مثلث های موجود در هر مثلث احاطه شده ؟
کلا تعدا مثلث هاییکه میتونیم تو یه مثلث دیگه بکشیم چند تاست؟
بی نهایته ؟یه کسی میگه جوابش میشه 4320؟؟؟؟؟؟؟؟؟:n36:

سلام دوست عزیز
دلیلی نداره که تعداد مثلث ها محدود باشن ، همون طور که یه پاره خط بی نهایت نقطه داره.
یکی از راس ها رو میشه روی یک ضلع مثلت در نظر گرفت که خودش بی نهایت نقطه داره.
بی نهایته:n01:

marry77
01-01-2015, 13:18
پاره خط هیچ محدودیتی نداره
اما ما اینو میدونیم که مثلث محدودیتی داره و مجموعه زوایای داخلیش میشه 180 پس یک جا این مثلث ها تموم میشه ،

chekmate
02-01-2015, 17:55
پاره خط هیچ محدودیتی نداره
اما ما اینو میدونیم که مثلث محدودیتی داره و مجموعه زوایای داخلیش میشه 180 پس یک جا این مثلث ها تموم میشه ،
مجموع زوایای داخلی مثلث ارتباطی با این سوالی که شما مطرح کردین، نداره. جواب هم همون بینهایت هست که دوست مون گفتن.

ArA_98
14-01-2015, 12:59
لطفا این رو اثبات کنید
مننون میشم
مجموع اندازه سه میانه هر مثلث از 4\3(سه چهارم)مجموع اندازه های سه ضلع آن مثلث بزرگتر است

tashant
15-10-2015, 11:49
سلام، 3تا قضیه هستش که لطفا اثباتش کنین:)
1)ثابت کنین در هر مثلث زاویه ی خارجی برابر است با مجموع زوایای داخلی غیر مجاور
2)ثابت کنین مجموع زوایا n ضلعی محدب،360 درجه است
3)ثابت کنین عمود منصف ها همرس هستند

paveej
15-10-2015, 16:34
سلام، 3تا قضیه هستش که لطفا اثباتش کنین:)
1)ثابت کنین در هر مثلث زاویه ی خارجی برابر است با مجموع زوایای داخلی غیر مجاور
2)ثابت کنین مجموع زوایا n ضلعی محدب،360 درجه است
3)ثابت کنین عمود منصف ها همرس هستند
سلام
1-از يه زاويه خارجي يه خط به موازات ضلع مقابلش رسم ميكنيم و با استفاده از تساوي زاويه ها در خطوط موازي قضيه ثابت ميشه.
A=C1
B=C2
C=C1+C2

نتيجه ميگيريم : C=A+B

[ برای مشاهده لینک ، لطفا با نام کاربری خود وارد شوید یا ثبت نام کنید ] ([ برای مشاهده لینک ، لطفا با نام کاربری خود وارد شوید یا ثبت نام کنید ])

2-با توجه به شكل زير ميبينيم كه سمت راست هر قطر كه از راس A عبور ميكنه يه مثلث تشكيل ميشه.چون تعداد قطرهاي عبوري از هر راس n ضلعي برابر n-3 ميشه
بنابراين n-3 مثلث تشكيل ميشه.يه مثلث هم سمت چپ آخرين قطر تشكيل ميشه كه تعداد مثلثها ميشه n-2 .بنابراين مجموع زواياي داخلي n ضلعي محدب ميشه
حاصلضرب n-2 در 180

[ برای مشاهده لینک ، لطفا با نام کاربری خود وارد شوید یا ثبت نام کنید ] ([ برای مشاهده لینک ، لطفا با نام کاربری خود وارد شوید یا ثبت نام کنید ])

یاسی1100
13-11-2015, 15:06
سلام میشه این سوال من حل کنید؟ ممنون میشم
از مثلث متساوی الساقینی سه ارتفاع آن معلوم است .ان را رسم کنید
فقط سریع لطفا

paveej
13-11-2015, 23:04
سلام میشه این سوال من حل کنید؟ ممنون میشم
از مثلث متساوی الساقینی سه ارتفاع آن معلوم است .ان را رسم کنید
فقط سریع لطفا

از راس A خطي بموازات ارتفاع AH رسم ميكنيم تا امتداد ضلع AC را در نقطه P قطع كند.از A به موازات BC خطي رسم ميكنيم تا BP را در نقطه K قطع كند.مثلثهاي AKP و AKB به حالت (زض ز) با هم مساويند.
با توجه به روابط گفته شده در شكل مثلث 'BPH با داشتن وتر BPو يك ضلع زاويه قائمه'BH قابل رسم است.

بعد از رسم اين مثلث ضلع H'P را ازسمت زاويه قائمه يعني 'H امتداد ميدهيم و خطي در نقطه B عمود بر ضلع BP رسم ميكنيم. راس C محل برخورد اين دو خط خواهد بود.
و خطي از نقطه K وسط BP عمود بر BP رسم ميكنيم تا 'PH را در نقطه A قطع كند.


[ برای مشاهده لینک ، لطفا با نام کاربری خود وارد شوید یا ثبت نام کنید ]

([ برای مشاهده لینک ، لطفا با نام کاربری خود وارد شوید یا ثبت نام کنید ])


براي رسم مثلث 'BPH ابتدا وتر BP را به اندازه 2AH رسم كرده سپس دايره اي به شعاع نصف وتر و به مركز وسط وتر رسم ميكنيم.سپس دايره ديگري به شعاع 'BH و مركز B
رسم ميكنيم تا يكديگر را در نقطه 'H قطع كنند.

یاسی1100
16-11-2015, 23:28
مرسی واقعا .قداتون

nima1401
04-05-2016, 14:21
سلام دوستان عزیز من اثبات90.( 2n-4(مجموع زاویه های داخلی هر nضلعی محدب)رو میخوام خواهش میکنم هر چه سریعتر به من کمک کنید

سلام دوست عزیز کاری نداره تو پرانتز از2 فاکتور که بگیری میشه90*2(2n-4)خب حالا اگه چون فاکتور گرفتی بین 2 و عبارت داخل پرانتز و 90 ضرب هستش پس 2*90 که بشه میشه 180 حالا عبارت اصلی اینه 180*(n-2) که اینم بدیهیه مثلث 3تا ضلع داره مجموع زاویه های داخلیش 180 درجه است

محمد7966
04-05-2016, 14:28
سلام دوست عزیز کاری نداره تو پرانتز از2 فاکتور که بگیری میشه90*2(2n-4)خب حالا اگه چون فاکتور گرفتی بین 2 و عبارت داخل پرانتز و 90 ضرب هستش پس 2*90 که بشه میشه 180 حالا عبارت اصلی اینه 180*(n-2) که اینم بدیهیه مثلث 3تا ضلع داره مجموع زاویه های داخلیش 180 درجه است

سلام

خوبید؟

حواستون باشه هر وقت جواب کسی رو میخاین بدید به تاریح پست هم توجه کنید

البته ایرادی نداره منم اولش عین شماا بودم،حواسم نبود:n02:

nima1401
04-05-2016, 14:32
سلام دوست عزیز کاری نداره تو پرانتز از2 فاکتور که بگیری میشه90*2(2n-4)خب حالا اگه چون فاکتور گرفتی بین 2 و عبارت داخل پرانتز و 90 ضرب هستش پس 2*90 که بشه میشه 180 حالا عبارت اصلی اینه 180*(n-2) که اینم بدیهیه مثلث 3تا ضلع داره مجموع زاویه های داخلیش 180 درجه است

nima1401
04-05-2016, 14:33
سلام اره حواسم نبود سوالات اینا خیلی الکیه:n02::n02:

حلما 91
17-11-2016, 18:11
سلام
کسی می تونه این دوتا گزاره رو حل کنه (از بنداشت هیلبرت)
گزاره:اگرA*B*C داده شده باشد آنگاه Bتنها نقطه مشترک میان نیم خط های BAوBCاست وAB=AC
گزاره:الف)اگر نیم خطr که از نقطه ای واقع در بیرون مثلت ABC رسم می شود ضلع AB را در نقطه ای بینAوB ببرد .ضلع ACیا ضلع BCرا هم می برد
ب)لگر نیم خطی از یک نقطه ی واقع در درون مثلث ABC رسم شود یکی از اضلاع را می برد و اگر از یکس از رئوس نگذرد تنها یکی از ضلعها را می برد.

حلما 91
17-11-2016, 19:16
سلام
کسی می تونه این دوتا گزاره رو حل کنه (از بنداشت هیلبرت)
گزاره:اگرA*B*C داده شده باشد آنگاه Bتنها نقطه مشترک میان نیم خط های BAوBCاست وAB=AC
گزاره:الف)اگر نیم خطr که از نقطه ای واقع در بیرون..